ENVIRONMENT

Previous Year Questions

www.laex.in Page No. 1 https://elearn.laex.in

www.laex.in

ENVIRONMENT

Previous Year Questions

www.laex.in Page No. 1 https://elearn.laex.in

www.laex.in

Environment UPSC

Previous Year Questions

ENVIRONMENT UPSC PREVIOUS YEAR QUESTIONS

INDEX

No. of Page No. S.No. Topic Qns From To

01. Act and Policies 13 1 12

02. Organization /Bodies/ NGOs 7 12 16

03. Bio-Diversity 50 17 37

04. Animal Bio-Diversity 48 37 53

05. Ecology and Environment 21 55 64

06. Global Environmental Issues 9 64 68

07. Global Level Efforts to Fight Climate Change 20 69 79

08. Pollution 14 79 85

www.laex.in https://elearn.laex.in

n

www.laex.in

Environment UPSC

Previous Year Questions

I Act and Policies (b) The Rules are applicable to notified urban 1. Pollution Related local bodies, notified towns and all industrial townships only. 1.1 Environment Protection Act 1986 (c) The rules provides for exact and elaborate 1. Consider the following statements: criteria for the identification of sites for The Environment Protection Act, 1986 landfills and waste processing facilities empowers the Government of to (d) It is mandatory on the part of waste 1) State the requirement of public generator that the waste generated in one participation in the process of district cannot be moved to another environmental protection, and the district. procedure and manner in which it is 1.3 E-Waste sought. (Management and Handling) Rules 2011 2) Lay down the standards for emission or

discharge of environmental pollutants from 4. In India, 'extended producer various sources. responsibility' was introduced as an Which of the statements given above is/are important feature in which of the correct? following? (a) 1 only a. The Bio-medical Waste (Management and Handling)Rules,1998 (b) 2 only b. The Recycled Plastic (Manufacturing and (c) Both 1 and 2 Usage)Rules,1999 (d) Neither 1 nor 2 c. The e-Waste (Management and Handling)

2. The Genetic Engineering Appraisal Rules, 2011 Committee is constituted under the d. The Food Safety and Standard Regulations, (a) Food Safety and Standards Act, 2006 2011 (b) Geographical Indications of Goods 1.4 National Green Tribunal Act (Registration and Protection) Act, 1999 2010

(c) Environment (Protection) Act, 1986 5. The National Green Tribunal Act, 2010 (d) Wildlife (Protection) Act, 1972

was enacted in consonance with which 1.2 Solid Waste Management of the following provisions of the Rules, 2016 Constitution of India? 3. As per the Solid Waste Management 1) Right to healthy environment, construed as Rules, 2016in India, which one of the a part of Right to life under Article 21. following statements is correct? 2) Provision of grants for raising the level of (a) Waste generator has to segregate waste administration in the Scheduled Areas for into five categories.

www.laex.in Page No. 1 https://elearn.laex.in

n

www.laex.in

Environment UPSC

Previous Year Questions

the welfare of Scheduled Tribes under d. Both (b) and (c) stated above are correct in Article 275(1). this context. Which of the statements given above is/are correct? 2.2 The Scheduled Tribes and other a. 1 only Traditional Forest Dwellers (Recognition b. 2 only of Forest Rights) Act, 2006 c. Both 1 and 2 8. Under the Scheduled Tribes and Other d. Neither 1 nor 2 Traditional Forest Dwellers (Recognition

2. Conservation Related of Forest Rights) Act, 2006, who shall be 2.1 Wildlife (Protection) Act, 1972 the authority to initiate the process for determining the nature and extent of 6. According to the Wildlife (Protection) individual or community forest rights or Act, 1972, which of the following both? animals cannot be hunted by any person a. State Forest Department except under some provisions provided b. District Collector/Deputy Commissioner by law? c. Tahsildar/Block Development Officer 1) Gharial /Mandal Revenue Officer 2) Indian wild ass d. Gram Sabha

3) Wild buffalo Select the correct answer using the code 9. Consider the following statements: given below 1) The definition of "Critical Wildlife Habitat" a. 1 only is incorporated in the Forest Rights Act, b. 2 and 3 only 2006. c. 1 and 3 only 2) For the first time in India, Biogas has been d. 1,2 and 3 given Habitat Rights. 3) Union Ministry of Environment, Forest and 7. In India, if a species of tortoise is Climate Change officially decides and declared protected under Schedule I of declares Habitat Rights for Primitive and the Wildlife (Protection) Act, 1972, what Vulnerable Tribal Groups in any part of does it imply? India. a. It enjoys the same level of protection as the Which of the statements given above is/are tiger. correct? b. It no longer exists in the wild, a few a. 1 and 2 only individuals are under captive protection; b. 2 and 3 only and now it is impossible to prevent its c. 3 only extinction. d. 1, 2 and 3 c. It is endemic to a particular region of India.

www.laex.in Page No. 2 https://elearn.laex.in

n

www.laex.in

Environment UPSC

Previous Year Questions

10. Consider the following statements: a. 1 and 2 only 1) As per recent amendment to the Indian b. 2 and 3 only Forest Act, 1927, forest dwellers have the c. 3 only right to fell the bamboos grown on the d. 1,2 and 3 forest areas. 3. Water 2) As per the Scheduled Tribes and Other 12. If National Water Mission is properly Traditional Forest Dwellers (Recognition of and completely implemented, howwill it Forest Rights) Act, 2006, bamboo is a impact the country? minor forest produce. 1. Part of the water needs of urban areas will 3) The Scheduled Tribes and Other be met through recycling of waste-water. Traditional Forest Dwellers (Recognition of 2. The water requirements of coastal cities Forest Rights) Act, 2003 allows ownership with inadequate alternative sources of of minor forest produce to forest dwellers. water will be met by adopting appropriate Which of the statements given above is/are technologies that allow for the use of ocean correct? water a. 1 and 2 only 3. All the rivers of Himalayan origin will be b. 2 and 3 only linked to the rivers of peninsular India c. 3 only 4. The expenses incurred by farmers for d. 1, 2 and 3 digging bore- wells and for installing

2.3 Wetlands motors and pump-sets to draw ground- (Conservation and Management) Rules, water will be completely reimbursed by the 2010 Government 11. Consider the following statements: Select the correct answer using the codes 1) Under Ramsar convention, it is mandatory a. 1 only on the part of the government of India to b. 1 and 2 only protect and conserve all the wetlands in c. 3 and 4 only the territory of India. d. 1, 2, 3 and 4 2) The wetlands (conservation and 4. Miscellaneous management) rules, 2010 were framed by 13. With reference to India, consider the Government of India based on the following Central Acts: recommendations of Ramsar convention. 1) Import and Export (Control) Act, 1947 3) The wetlands (Conservation and 2) Mining and Mineral Development (Regulation) Management) Rules, 2010 also encompass Act, 1957 the drainage area or catchment regions of 3) Customs Act, 1962 the wetlands as determined by the 4) Indian Forest Act, 1927 authority Which of the above Acts have relevance Which of the statements given above is/are to/bearing on the biodiversity conservation in correct? the country?

www.laex.in Page No. 3 https://elearn.laex.in

n

www.laex.in

Environment UPSC

Previous Year Questions

a. 1 and 3 only environment. Hence, all the Environmental b. 2, 3 and 4 only legislations of Indian parliament becomes c. 1, 2, 3 and 4 important. One such act is Environmental d. None of the above Acts Protection Act of 1986. Key and Explanations 2. Key: C

Explanation: Genetic Engineering Appraisal 1. Pollution Related Committee 1.1 Environment Protection Act 1986 The Genetic Engineering Appraisal Committee

1. Key: B (GEAC) was constituted under the Ministry of Explanation: Environment Protection Act Environment, Forest and Climate Change 1986 (MoEFCC) as the apex body under the ‘Rules for  In the wake of the Bhopal tragedy, the Manufacture, Use, Import, Export and Storage of government of India enacted the Hazardous Microorganisms/Genetically Environment (Protection) Act of 1986. Engineered Organisms or Cells 1989’ in  To protect and improve environmental accordance with the Environment Protection qualities. Act, 1986.  To establish an authority to study, plan Hence answer is option C and implement long term requirements of About GEAC environmental safety.  The ministry’s Special Secretary/Additional Section 3: Central Government shall have the Secretary is the Chairman of the GEAC. power to take all such measures as it deems  Currently, the GEAC has twenty-four necessary or expedient for the purpose of members and meets once every month to protecting and improving the quality of the carry out its prescribed activities. environment and preventing, controlling and Functions abating environmental pollution. To appraise activities involving the large-scale The act does not mention about requirement use of hazardous microorganisms and of public participation. Hence option 1 is recombinants in research and industrial wrong. (however, sec 3 can be close to option production from the environmental angle. 1 as it empowers central government to do To appraise proposals relating to release of so. Hence need to wait for the official key) genetically engineered organisms and Section 3-2 (iv) Central Government may lay products into the environment including down standards for emission or discharge of experimental field trials. environmental pollutants from various The committee or any persons authorized by it sources whatsoever. Hence option 2 is has powers to take punitive action under the correct Environment Protection Act. Educational Objective: Indian constitution Educational Objective: to have knowledge have very few provisions relating to the regarding organisations that deal with Environmental laws. And also to know from

www.laex.in Page No. 4 https://elearn.laex.in

n

www.laex.in

Environment UPSC

Previous Year Questions

where the organisation derive powers from. There is no mention of district waste Whether statutory or executive. management requirement, so D is wrong. a) The Rules provide for exact and elaborate Educational Objective: To know the provisions criteria for the identification of sites for of various pollution related rules and laws like landfills and waste processing facilities. plastic waste management rules, construction b) It is mandatory on the part of waste and demolition waste management rules, e generator that thewaste generated in one waste management rules etc.

district cannot be moved to another district. 1.3 E-Waste (Management and

1.2 Solid Waste Management Handling) Rules 2011

Rules, 2016 4. Key: C 3. Key: C Explanation: E-Waste (Management) Rules, Explanation: Solid Waste Management Rules 2016 2016 The Ministry of Environment, Forest and These rules replace the Municipal Solid Wastes Climate Change notified the E-Waste (Management and Handling) Rules, 2000, are Management Rules, 2016 in supersession of the now applicable beyond municipal areas and e-waste (Management & Handling) Rules, 2011. have included urban agglomerations, census Making the norms stringent, the new E-waste towns, notified industrial townships etc. rules included Compact Fluorescent Lamp (CFL) Hence option b is wrong and other mercury containing lamps, as well as The new rules have mandated the source other such equipments. segregation of waste in order to channelize the For the first time, the rules brought the waste to wealth by recovery, reuse and recycle. producers under Extended Producer Waste generators would now have to now Responsibility (EPR), along with targets. segregate waste into three streams (hence Hence option C is correct. option a is wrong)- Biodegradables, Dry Producers have been made responsible for (Plastic, Paper, metal, Wood, etc.) and Domestic collection of E-waste and for its exchange. Hazardous waste (diapers, napkins, mosquito Various producers can have a separate Producer repellants, cleaning agents etc.) before handing Responsibility Organisation (PRO) and ensure it over to the collector. collection of E-waste, as well as its disposal in Rules specifies criteria for site selection, an environmentally sound manner. development of facilities at the sanitary landfills, Educational Objective: to have knowledge specifications for land filling operations and about the highlights of all the waste closure on completion of land filling, pollution management Rules like plastic waste, e waste, prevention, Closure and Rehabilitation of Old solid waste, construction and demolition waste Dumps, specifies Criteria for special provisions etc. for hilly areas. So, C is correct.

www.laex.in Page No. 5 https://elearn.laex.in

n

www.laex.in

Environment UPSC

Previous Year Questions

1.4 National Green Hence option 1 is correct. Tribunal Act 2010 Article 275(1) deals with the grants to the states. The funds are released to the State Governments 5. Key: A against specific projects for the welfare of Explanation: Article 21 is the most widely Scheduled Tribes and strengthening of interpreted Article in the Indian Constitution. administration of tribal areas. Supreme Court in number of cases including Here Option 2 is correct but NGT ACT IS NOT Menaka Gandhi Casehave widened the scope of enacted in consonance with Article 275(1). the article to include many natural rights in it. Hence option 2 in this question is wrong. One such right is Right to healthy Environment. Educational Objective: To know about the Parliament of India have enacted NGT Act 2010, important legislations relating to the in order to implement the provisions of this Environment and their relation provisions in the article. constitution of India.

www.laex.in Page No. 6 https://elearn.laex.in

n

www.laex.in

Environment UPSC

Previous Year Questions

www.laex.in Page No. 7 https://elearn.laex.in

n

www.laex.in

Environment UPSC

Previous Year Questions

Iucn Conservation Name And Image Habitat Threats Status Status Ramganga, River

loss of riverine habitat, Girwa, River CITES depletion of fish Gandaki, River Gharial Appendix I, resources, and Chambal, River Schedule I of entanglement in fishing Ganges,Son River, Wildlife nets. Gharials were killed Mahanadi River in (Protection) by fishermen, hunted for Odisha.Nepal: Rapti- Act 1972. skins, trophies and Narayani River, CriticallyEndangered indigenous medicine. Bangladesh is found

predominantly in the Indian Wild Ass Schedule I of Little Rann of Kutch The animal was also Wildlife and its surrounding harvested for meat in (Protection) areas in . India in the past Act 1972. Endangered Preferably in saline deserts is mainly found in the alluvial

Wild Buffalo grasslands, marshes, hunting, especially in Schedule I of (BubalusArnee) swamps and river Thailand, Cambodia, and Wildlife valleys. They are Myanmar; (Protection) generally found in habitat loss of floodplain . Act 1972. Endangered areas that have degradation of wetlands plenty of water holes and resources

www.laex.in Page No. 8 https://elearn.laex.in

n

www.laex.in

Environment UPSC

Previous Year Questions

www.laex.in Page No. 9 https://elearn.laex.in

n

www.laex.in

Environment UPSC

Previous Year Questions

2. Conservation Related Educational Objective: To have idea about the 2.1 Wildlife (Protection) Act, 1972 degree of protection that various animals enjoy under Wild Life Protection Act, 1972.

6. Key: D Explanation: All the three animals come under 2.2 The Scheduled Tribes and other Schedule I of Wildlife (Protection) Act 1972. Traditional Forest Dwellers (Recognition Hence, these are given absolute protection from of Forest Rights) Act, 2006 hunting, except under some provisions provided 8. Key: D by law. Explanation: Forest Rights Act, 2006 Educational Objective: to have knowledge abot The act recognize and vest the forest rights and the conservation status of various animals occupation in Forest land in forest Dwelling thatare often seen in news.

Scheduled Tribes (FDST) and Other Traditional 7. Key: A Forest Dwellers (OTFD)who have been residing Explanation: Wild Life Protection Act (Wpa) – in such forests for generations Schedules It identifies 4 rights Tiger in India is placed under Schedule I of 1) Title rights Wildlife (Protection) Act 1972. 2) Use rights The act provides for six schedules to regulate the 3) Relief and development rights trade on animals. 4) Forest management rights Schedule Animals which are in the Allocation of these rights I and part category of endangered species. Gram sabha (full village assembly, NOT the II of These are given absolute gram panchayat) makes a recommendation – schedule protection from hunting. Eg i.e., who has been cultivating land for how long, II Tiger which minor forest produce is collected, etc. These also have roughly the Hence, answer is option D Schedule same provisions of Section I and Educational Objective: Tribes and their rights III and IV II, but cover animals that are not is one of the hot topics for UPSC. In this regard in danger of becoming extinct. everything about tribes becomes important fo Delineates animals that can be the exam Schedule hunted like ducks and deers 9. Key: A V with the prior permission of Explanation: The Critical Wildlife Habitats have chief wildlife warden. (Vermins) been envisaged in Scheduled Tribes and Other Concerns cultivation and plant Schedule Traditional Forest Dwellers (Recognition of life and gives teeth to setting up VI Forest Rights) Act, 2006.Hence option 1 is more protected animal parks. right.

In a bid to undo historical injustice meted out to primitive tribal communities living in central

www.laex.in Page No. 10 https://elearn.laex.in

n

www.laex.in

Environment UPSC

Previous Year Questions

India, the government of Madhya Pradesh has bamboo is a minor forest produce as per the act. for the first time recognised the habitat rights of Hence option 2 and 3 is correct. seven villages in Dindori district, mostly Educational Objective: To have knowledge inhabited by the Baiga. about the rights of tribes as per the forest Baigatribals became India's first community to dwellers rights Act and the recent amendment get habitat rights. ... The tribe numbers only brought to it. 150,000 people spread over forested areas of 2.3 Wetlands Madhya Pradesh and Chhattisgarh. For several (Conservation and Management Rules decades, the Baigas were discriminated against 2010) and often evicted from forest areas by 11. Key: C government agencies. Option 2 is correct. Explanation: Ramsar Convention The Ministry of Tribal Affairs is implementing It is an international treaty for “the conservation the ―Scheme of Development of Particularly and sustainable use of wetlands”. It is also Vulnerable Tribal Groups (PVTGs). It aims at known as the Convention on Wetlands. planning their socio-economic development in a Ramsar philosophy is “wise use” of wetlands. comprehensive manner while retaining the At the time of joining the Convention, each culture and heritage of the community. Hence Contracting Party undertakes to designate at option 3 is wrong least one wetland site (not all the sites. Educational Objective: Forest Rights Act is Hence option 1 is wrong) for inclusion in the one of the landmark legislations in India for List of Wetlands of International Importance. granting rights to the tribes. Hence all the The inclusion of a “Ramsar Site” in the List provisions of the act are important for the exam embodies the government’s commitment to take

10. Key: B the steps necessary to ensure that its ecological Explanation: The Parliament brought an character is maintained. amendment to exclude bamboo from the The Ministry of Environment and Forests, definition of tree under the Indian Forest recognising the importance of having a legal Act, claiming it would improve the earnings of framework for the preservation and tribals and dwellers living around forests. management of wetlands in India, drew up a The amendment permits felling and transit of draft Regulatory Framework drawing upon the bamboo grown in non-forest areas. However, recommendation of a wide spectrum of bamboo grown on forest lands would continue experts and stakeholders. to be classified as a tree and would be guided These rules were not based on Ramsar by the existing legal restrictions. Hence Convention. Hence option 2 is wrong option 1 is wrong. Educational Objective: wetlands are the most The Scheduled Tribes and Other Traditional important ecosystems that help in large Carbon Forest Dwellers Act, 2003 allows ownership of Sink. Hence conservation efforts towards minor forest produce to forest dwellers and

www.laex.in Page No. 11 https://elearn.laex.in

n

www.laex.in

Environment UPSC

Previous Year Questions

wetlands both national and international are While Option 2 and 4 are very certain to deal important for the examination point of view. with the biodiversity, the other two options also

3. Water deal with Biodiversity indirectly. E.g both the acts deal with entering of Alien Species 12. Key: B (Invasive Species) from other countries. Explanation: National Water Mission: Educational Objective: To have knowledge It is one of the eight missions launched under about various Environmental Legislations of the National Action Plan on Climate Change Indian Parliament that deals with conservation (NAPCC) for combating the threats of global of Nature. warming.

Objectives of the National Water Mission are, II. Organizations/Bodies/NGO’s Increase water use efficiency by 20% through

regulations, differential entitlements and pricing. 1. Pollution Related A considerable share of water needs of urban 1.1 National Green Tribunal & CPCB areas is to be met through recycling of wastewater. Hence statement 1 is correct 1. How is the National Green Tribunal Water requirements of coastal cities are to be (NGT) different from the Central met through the adoption of low-temperature Pollution Control Board (CPCB)? desalination technologies. Hence, statement 2 1) The NGT has been established by an Act is correct whereas the CPCB has been created by an Consult with states to ensure that basin-level executive order of the Government. management strategies are made to deal with 2) The NGT provides environmental justice variability in rainfall and river flows due to and helps reduce the burden of litigation in climate change. the higher courts whereas the CPCB Enhance storage above and below ground, promotes cleanliness of streams and wells, implement rainwater harvesting. and aims to improve the quality of air in Adopt large scale irrigation programmes which the country. rely on sprinklers, drip irrigation and ridge and Which of the statements given above is/are furrow irrigation. correct? Educational Objective: To know about impact a. 1 only of National Water Mission. b. 2 only c. Both 1 and 2 4. Miscellaneous d. Neither 1 nor 2 13. Key: C 1.2 Biodiesel Mission – Explanation: Totally there 35 Acts that are legislated by the Indian Parliament that deal Implementing Ministry

with Biodiversity in one or the other way. All the 2. Which one of the following Union above 4 Acts Deal with the conservation of Ministries is implementing the Biodiversity. Biodiesel Mission (as Nodal Ministry)?

www.laex.in Page No. 12 https://elearn.laex.in

n

www.laex.in

Environment UPSC

Previous Year Questions

a. Ministry of Agriculture Which of the statements given above is/are b. Ministry of Science and Technology correct? c. Ministry of New and Renewable Energy a. 1 and 3 only d. Ministry of Rural Development b. 2 only

2. Conservation Related c. 2 and 3 only 2.1 National Biodiversity Authority d. 1,2 and 3

3. How does National Biodiversity 2.3 Gadgil + Kasthuri Authority (NBA) help in protecting the Rangan Committee

Indian agriculture? 5. ‘Gadgil Committee Report’ and 1) NBA checks the biopiracy and protects the ‘Kasturirangan Committee Report’, indigenous and traditional genetic sometimes seen in the news, are related resources. 2) NBA directly monitors and supervises the to scientific research on genetic modification a. Constitutional reforms of crop plants. b. Ganga Action Plan 3) Applicati0on for Intellectual Property Rights c. Linking of rivers related to genetic/biological resources cannot d. Protection of be made without the approval of NBA. 2.4 National Ganga RiverBasin Which of the statements given above is/are Authority’ (NGRBA) correct? a. 1 only 6. Which of the following are the key b. 2 and 3 only features of ‘National Ganga River Basin c. 1 and 3 only Authority’ (NGRBA)? d. 1,2 and 3 1) River basin is the unit of planning and management. 2.2 Bombay Natural 2) It spearheads the river conservation efforts History Society NGO at the national level.

4. With reference to Bombay Natural 3) One of the Chief Ministers of the States History Society (BNHS), consider the through which the Ganga flows becomes following statements: the Chairman of NGRBA on rotation basis. 1) It is an autonomous organization under the Select the correct answer using the code given below. Ministry of Environment and Forests. a. 1 and 2 only 2) It strives to conserve nature through b. 2 and 3 only action-based research, education and c. 1 and 3 only public awareness. d. 1, 2 and 3 3) It organizes and conducts nature trails and

camps for the general public.

www.laex.in Page No. 13 https://elearn.laex.in

n

www.laex.in

Environment UPSC

Previous Year Questions

2.5 Miscellaneous Act, 1974, and the Air (Prevention and Control of 7. Consider the following statements: Pollution) Act, 1981, to 1) Animal Welfare Board of India is (i) Promote cleanliness of streams and established under the Environment wells in different areas of the States by (Protection) Act, 1986. prevention, control and abatement of water 2) National Tiger Conservation Authority is a pollution, and statutory body. (ii) Improve the quality of air and to 3) National Ganga River Basin Authority is prevent, control or abate air pollution in chaired by the Prime Minister. the country. Which of the statements given above is / are Hence option 2 is correct correct? Educational Objective: to have knowledge a. 1 only about the functions of various b. 2 and 3 only environmental organisations in India. c. 2 only 1.2 Biodiesel Mission – d. 1, 2 and 3 Implementing Ministry

Key and Explanations 2. Key: D Explanation: Biodiesel Mission was launched by

1. Pollution Related Government of India in 2007 to encourage the 1.1 National Green Tribunal & CPCB use of biodiesel. Ministry of Rural Development

was made as the nodal ministry for the 1. Key: B implementation of this mission Explanation: Both NGT and CPCB are Government of India has comeup with National statutory bodies i.e. established by acts Biofuel Policy In 2018. NGT is established by NGT ACT itself where as CPCB, is statutory organisation, was constituted in 1974 under the Water (Prevention and Control of Pollution) Act, 1974. Hence option 1 is wrong One of the main objectives of establishing NGT: The Tribunal’s dedicated jurisdiction in environmental matters shall provide speedy environmental justice and help reduce the burden of litigation in the higher courts. Educational Objective: To have idea about Principal functions of the CPCB, as spelt out in various policies and Missions of the Government the Water (Prevention and Control of Pollution) and about their implementation ministries.

www.laex.in Page No. 14 https://elearn.laex.in

n

www.laex.in

Environment UPSC

Previous Year Questions

2. Conservation Related  It supports many research efforts through 2.1 National Biodiversity Authority grants and publishes the Journal of the Bombay Natural History Society. 3. Key: D  It organizes and conducts nature trails Explanation: National Biodiversity Authority. and camps for the general public.(option  Established in 2003 to implement India’s 3 is correct) Biological Diversity Act (2002)  Department of Science and Technology has  Statutory, Autonomous Body designated it as a ‘Scientific and  Headquarters: Chennai, Industrial Research Organisation’. Functions  BNHS is the partner of Bird Life  The Act covers conservation, use of International in India. biological resources and associated Educational Objective: To know about various knowledge occurring in India for NGO’s that are working towards the commercial or research purposes or for conservation of nature in India and across the the purposes of bio-survey and bio- world. utilisation. 2.3 Gadgil + Kasthuri  It provides a framework for access to Rangan Committee biological resources and sharing the

benefits arising out of such access and 5. Key: D use. Explanation: Both the committees are related to  The Act also includes in its ambit the protection of western ghats by declaring transfer of research results and application Ecologically Sensitive Area. Hence option D is for intellectual property rights (IPRs) correct. relating to Indian biological resources. Educational Objective: to have idea about Educational Objective: to have knowledge various committees that are setup to protect the about functions and duties of Statutory environment and also to have knowledge about environmental organisations in India. important recommendations of those committees

2.2 Bombay Natural History Society NGO 4. Key: C Explanation: BNHS is one of the largest non- governmental organization (NGO) in India engaged in conservation of nature through action- based research, education and public awareness. Not a government Institution. Hence option 1 is wrong and option 2 is correct It was founded on 15 September 1883 and headquartered at Hornbill House, Mumbai.

www.laex.in Page No. 15 https://elearn.laex.in

n

www.laex.in

Environment UPSC

Previous Year Questions

2.4 National Ganga River Basin Cruelty to Animals Act,1960. Hence 1 is Authority’ (NGRBA) wrong  Under Ministry of Environment and 6. Key: A Forests Explanation: ‘National Ganga River Basin  HQ Ballabhgarh in Faridabad District Authority’ (NGRBA)  It is an advisory body It frames a range of Constituted under Environment (Protection) Act, rules on how animals ought to be 1986 The Ministry of Water Resources, River humanely treated everywhere. Development and Ganga Rejuvenation (MoWR, National Ganga River Basin Authority RD & GR) is the nodal Ministry for the NGRBA  Constituted under Section 3(3) of the Chaired by the Prime Minister and has as its Environment (Protection) Act, 1986 members the Union Ministers concerned, the  The Ministry of Water Resources, River Chief Ministers of the States through which Development and Ganga Rejuvenation Ganga flows (hence option 3 is wrong, by (MoWR, RD & GR) is the nodal Ministry for elimination answer is option a) the NGRBA Functions  Chaired by the Prime Minister and has  It is a planning, financing, monitoring as its members the Union Ministers and coordinating body of the centre and concerned, the Chief Ministers of the the states States through which Ganga flows. Hence option 3 is correct  Development of a river basin management National Tiger Conservation Authority plan It is a statutory body under the Ministry of  Regulation of activities aimed at Environment, Forests and Climate Change prevention, control and abatement of constituted under enabling provisions of the pollution in Ganga to maintain its water Wildlife (Protection) Act, 1972, as amended in quality 2006, for strengthening tiger conservation, as  Maintenance of minimum ecological flows per powers and functions assigned to it under in the river Ganga the said Act. Hence option 2 is correct Educational Objective: To learn about various Educational Objective: to have knowledge initiatives towards the cleaning of River Ganga. about various environmental organisations that Also, about National Ganga Mission are striving to protect the environment.

2.5 Miscellaneous

7. Key: B

Animal Welfare Board of India

 It is a statutory body. Established in 1962

under Section 4 of The Prevention of

www.laex.in Page No. 16 https://elearn.laex.in

n

www.laex.in

Environment UPSC

Previous Year Questions

Burma regions as hotspots of III Bio-Diversity

biodiversity: 1. Basics 1) Species richness 1.1 Biodiversity Rich Areas 2) Vegetation density 1.1.1 Himalayan Range 3) Endemism

1. The Himalayan Range is very rich in 4) Ethno-botanical importance

species diversity. Which one among the 5) Threat perception following is the most appropriate reason 6) Adaptation of flora and fauna to warm and for this phenomenon? humid conditions a. It has a high rainfall that supports Which three of the above are correct criteria luxuriant vegetative growth. in this context? b. It is a confluence of different bio a. 1, 2 and 6 geographical zones. b. 2, 4 and 6 c. Exotic and invasive species have not been c. 1, 3 and 5 introduced in this region. d. 3, 4 and 6 d. It has less human interference. 1.2 Threats to Biodiversity

1.1. 2 Equatorial Forest 4. Due to some reasons, if there is a huge

2. Which of the following is/are unique fall in the population of species of characteristic/characteristics of butterflies, what could be its likely equatorial forests? consequence/consequences? 1) Presence of tall closely set trees with 1) Pollination of some plants could be crowns forming a continuous canopy. adversely affected. 2) Coexistence of a large number of species. 2) There could be a drastic increase in the 3) Presence of numerous varieties of fungal infections of some cultivated epiphytes. plants. Select the correct answer using the code 3) It could lead to a fall in the population of given below some species of wasps, spiders and a. 1 only birds. b. 2 and 3 only Select the correct answer using the code c. 1 and 3 only given below: d. 1, 2 and 3 a. 1 only

1.1. 3 Hot Spots Criteria b. 2 and 3 only

3. Three of the following criteria have c. 1 and 3 only contributed to the recognition of d. 1,2 and 3 Western Ghats-Sri Lanka and Indo-

www.laex.in Page No. 17 https://elearn.laex.in

n

www.laex.in

Environment UPSC

Previous Year Questions

5. Which of the following can be threats to 7. Other than resistance to pests, what are the biodiversity of a geographical area? the prospects for which genetically 1) Global warming engineered plants have been created? 2) Fragmentation of habitat 1. To enable them to withstand drought 3) Invasion of alien species 2. To increase the nutritive value of the 4) Promotion of vegetarianism produce Select the correct answer using the codes 3. To enable them to grow and do given below: photosynthesis in spaceships and space a. 1, 2 and 3 only stations b. 2 and 3 only 4. To increase their shelf life c. 1 and 4 only Select the correct answer using the codes d. 1, 2, 3 and 4 given below

6. The term "sixth mass extinction/sixth a. 1 and 2 only

extinction" is often mentioned in the b. 3 and 4 only

news in the context of the discussion of c. 1, 2 and 4 only a. Widespread monoculture practices in d. 1, 2, 3 and 4

agriculture and Large-scale commercial 1.3 Miscellaneous

farming with indiscriminate use of 8. Biodiversity forms the basis for human chemicals in many parts of the world that existence in the following ways: may result in the loss of good native 1) Soil formation ecosystems. 2) Prevention of soil erosion b. Fears of a possible collision of a meteorite 3) Recycling of waste with the Earth in the near future in the 4) Pollination of crops manner it happened 65 million years ago Select the correct answer using the codes that caused the mass extinction of many given below: species including those of dinosaurs. a. 1,2 and 3 only c. Large scale cultivation of genetically b. 2, 3 and 4 only modified crops in many parts of the world c. 1 and 4 only and promoting their cultivation in other d. 1, 2, 3 and 4 parts of the world which may cause the 9. Consider the following statements: disappearance of good native crop plants 1) Biodiversity is normally greater in the lower and the loss of food biodiversity. latitudes as compared to the higher d. Mankind's over-exploitation/misuse of latitudes. natural resources, fragmentation/loss of 2) Along the mountain gradients, biodiversity natural habitats, destruction of is normally greater in the lower altitudes as ecosystems, pollution and global climate compared to the higher altitudes. change.

www.laex.in Page No. 18 https://elearn.laex.in

n

www.laex.in

Environment UPSC

Previous Year Questions

Which of the statements given above is/are d. Wildlife Sanctuaries

correct? 13. Which one of the following National a. 1 only Parks lies completely in the temperate b. 2 only alpine zone? c. Both 1 and 2 a. Manas National Park d. Neither 1 nor 2 b. Namdapha National Park

10. Consider the following international c. Neora Valley National Park agreements: d. Valley of Flowers National Park 1. The International Treaty on Plant Genetic 14. Which one of the following National Resources for Food andAgriculture Parks has a climate that varies from 2. The United Nations Convention to Combat tropical to subtropical, temperate and Desertification arctic? 3. The World Heritage Convention a. Khangchendzonga National Park Which of the above has / have a bearing on b. Nandadevi National Park the biodiversity? c. Neora Valley National Park a. 1 and 2 only d. Namdapha National Park b. 3 only 15. Consider the following pairs: c. 1 and 3 only National Park River d. 1, 2 and 3 flowing through the Park 2. Protected Areas 1. Corbett National Park Ganga 2.1 National Park 2. Kaziranga National Park Manas 11. Which of the following National Parks is 3. Silent Valley National Park unique in being a swamp with floating Which of the above pairs is/are correctly vegetation that supports a rich matched? biodiversity? a. 1 and 2 a. Bhitarkanika National Park b. 3 only b. KeibulLamjao National Park c. 1 and 3 c. Keoladeo Ghana National Park d. None d. Sultanput National Park 16. Consider the following pairs:

12. In which one among the following Protected area Well-known for categories of protected areas in India 1. Bhiterkanika, Orissa Salt Water are local people not allowed to collect Crocodile and use the biomass? 2. Desert National Park, Great Indian a. Biosphere Reserves Rajasthan Bustard b. National Parks 3. Eravikulam, Hoolak Gibbon c. Wetlands declared under Ramsar Which of the pairs given above is/are Convention correctly matched?

www.laex.in Page No. 19 https://elearn.laex.in

n

www.laex.in

Environment UPSC

Previous Year Questions

a. 1 only 21. Recently there was a proposal to b. 1 and 2 only translocate some of the lions from c. 2 only their natural habitat in Gujarat to d. 1, 2 and 3 which one of the following sites?

17. Which one of the following is also known a. Corbett National Park as Top Slip? b. Kuno Palpur Wildlife Sanctuary a. Simlipal National Park c. Mudumalai Wildlife Sanctuary d. Sariska National Park b. Periyar Wildlife Sanctuary c. Manjira Wildlife Sanctuary 2.2 Tiger Reserve d. Indira Gandhi Wildlife Sanctuary and 22. From the ecological point of view, which National Park one of the following assumes importance

18. Which one of the following is located in in being a good link between the Eastern the Bastar region? Ghats and the Western Ghats? a. Bandhavgarh National Park a. Sathyamangalam Tiger Reserve b. Dandeli Sanctuary b. Nallamala Forest c. Rajaji National Park c. d. Indravati National Park d. Seshachalam Biosphere Reserve

19. Which one among the following has the 23. Consider the following protected areas maximum number of National Parks? 1) Bandipur a. Andaman & Nicobar Islands 2) Bhitarkanika b. Arunachal Pradesh 3) Manas c. Assam 4) Sunderbans d. Meghalaya Which of the above are declared Tiger

20. Consider the following statements Reserves? 1) Silent Valley National Park is in the a. 1 and 2 only Nallamalai range. b. 1, 3 and 4 only 2) Pathrakkadavu Hydroelectric Project is c. 2, 3 and 4 only proposed to be built near the Silent Valley d. 1, 2, 3 and 4 National Park. 2.3 Wildlife Sanctuaries

3) The Kunthi river originates in Silent Valley's 24. In which one of the following States is rainforests Pakhui Wildlife Sanctuary located? Which of the statements given above is/are a. Arunachal Pradesh correct b. Manipur a. 1 and 3 c. Meghalaya b. 2 only d. Nagaland c. 2 and 3 d. 1,2 and 3 25. The first marine sanctuary in India, having within its bounds coral reefs,

www.laex.in Page No. 20 https://elearn.laex.in

n

www.laex.in

Environment UPSC

Previous Year Questions

mollusca, dolphins, tortoises and various c. NandaDevi kinds of sea birds, has been established in d. Sundarbans a. Sundarbans 29. Match List-I (Biosphere Reserves) with b. Chilka Lake List-I (States) and select the correct c. Gulf of Kachchh answer using the codes given below the d. Lakshadweep

Lists: 2.4 Biosphere Reserve List-I List-II A. Similipal 1. Sikkim 26. The most important strategy for the B. DehongDeband 2. Uttaranchal conservation of biodiversity together C. Nokrek 3. Arunachal Pradesh with traditional human life is the D. Kanchenjunga 4. Orissa establishment of 5. Meghalaya a. Biosphere reserves Codes: b. Botanical gardens a. A-1; B-3; C-5; D-4 c. National parks b. A-4; B-5; C-2; D-1 d. Wildlife sanctuaries c. A-1; B-5; C-2; D-4

d. A-4; B-3; C-5; D-1 27. Which of the following are in

Agasthyamala Biosphere Reserve? 30. Which one of the following is not a a. , Peppara and Shendurney Wildlife Biosphere Reserve? Sanctuaries; and KalakadMundanthurai a. Agasthyamalai Tiger Reserve b. Nallamalai b. Mudumalai, Sathyamangalam and c. Nilgiri WayanadWildlife Sanctuaries; and Silent d. Panchmarhi

Valley National Park. 2.5 Eco-sensitive Zones c. Kaundinya, GundlaBrahmeswaram and 31. With reference to 'Eco-Sensitive Zones', Papikonda Wildlife Sanctuaries; and which of the following statements is/are National Park. correct? d. Kawal and Sri Venkateswara Wildlife 1) Eco-Sensitive Zones are the areas that are Sanctuaries; and Nagarjunasagar- declared under the Wildlife (Protection) Act, Srisailam Tiger Reserve

1972. 28. Out of all the Biosphere reserves in 2) The purpose of the declaration of Eco- India, four have been recognised on the Sensitive Zones is to prohibit all kinds of World Network by UNESCO human activities in those zones except Which one of the following is not one of agriculture. them? Select the correct answer using the code a. Gulf of Manner given below b. Kanchenjunga a. 1 only

www.laex.in Page No. 21 https://elearn.laex.in

n

www.laex.in

Environment UPSC

Previous Year Questions

b. 2 only c. 1 and 3 only c. Both 1 and 2 d. 1,2 and 3 d. Neither 1 nor 2 35. Match List I (National Park/Wildlife

2.6 Mix of above Categories Sanctuary) with List II (Nearby Town)

and select the correct answer using the 32. Consider the following pairs: code given below the lists: 1) Dampa Tiger Reserve Mizoram List-I List-II 2) Gumti Wildlife Sanctuary Sikkim A. Chandra Prabha 1. Jaipur 3) Saramati Peak Nagaland B. Kerera 2. Jhansi Which of the above pairs is/are correctly C. Jaisamand 3. Agra matched? D. Nahargarh 4. Varanasi a. 1 only 5. Udaipur b. 2 and 3 only Codes: c. 1 and 3 only a. A-4; B-1; C-5; D-2 d. 1,2 and 3

b. A-5; B-2; C-3; D-1 33. Consider the following pairs: c. A-4; B-2; C-5; D-1 1) Nokrek Biosphere Reserve Garo Hills d. A-5; B-1; C-3; D-2

2) Logtak (Loktak) Lake Barail Range 3) Namdapha National Park Dafla Hill 36. Match List-I (National Park/Sanctuary) Which of the above pairs is/are correctly with List-II (State) and select the correct matched? answer using the codes given below the a. 1 only Lists: b. 2 and 3 only List-I List-II A. Kanger Ghati 1. Chhattisgarh c. 1,2 and 3 National Park d. None

B. Nagerhole National 2. Haryana 34. Consider the following statements: Park 1) The boundaries of a National Park are C. Kugti Wildlife 3. Himachal Pradesh defined by legislation Sanctuary 2) A Biosphere Reserve is declared to D. Sultanpur Bird 4. conserve a few specific species of flora and Sanctuary fauna Codes: 3) In a Wildlife Sanctuary, limited biotic a. A-3; B-2; C-1; D-4 interference is permitted b. A-1; B-4; C-3; D-2 Which of the statements given above is/are c. A-3; B-4; C-1; D-2 correct? d. A-1; B-2; C-3; D-4 a. 1 only 37. Match List-I (National Park/Wildlife b. 2 and 3 only Sanctuary) with List-I1 (State) and

www.laex.in Page No. 22 https://elearn.laex.in

n

www.laex.in

Environment UPSC

Previous Year Questions

select the correct answer using the code Which of the statements given above is/are given below the lists: correct? List-I (National List-II a. 1 only Park/Wildlife (State) b. 2, 3 and 4 only Sanctuary) c. 1 and 3 only A. Bondla Wildlife 1. Orissa d. 1, 2, 3 and 4 Sanctuary B. Kangerghat 2. Assam 3.2 Insectivorous

National Park 40. Some species of plants are C. Orang Sanctuary 3. Chhattisgarh insectivorous. Why? D. Ushakothi Wildlife 4. a. Their growth in shady and dark places Sanctuary 5. Tripura does not allow them to undertake sufficient Codes: photosynthesis and thus they depend on a. A-2; B-1; C-5; D-3 insects for nutrition b. A-4; B-3; C-2; D-1 b. They are adapted to grow in nitrogen c. A-2; B-3; C-5; D-1 deficient soils and thus depend on insects d. A-4; B-1; C-2; D-3 for sufficient nitrogenous nutrition

3. Plant Diversity c. They cannot synthesize certain vitamins 3.1 Use/Importance of few Species themselves and depend on the insects digested by them 38. Recently, there was a growing awareness d. They have remained in that particular in our country about the importance of stage of evolution as living fossils, a link Himalayan nettle (Girardiniadiversifolia) between autotrophs and heterotrophs because it is found to be a sustainable 41. Which one of the following is an source of insectivorous plant? a. Anti-malarial drug a. Passion flower plant b. Biodiesel b. Pitcher plant c. Pulp for paper industry c. Night queen d. Textile fibre

d. Flame of the forest 39. Government of India encourages the cultivation of 'sea buckthorn'. What is 3.3 General Information of Individual the importance of this plant? Species

1) It helps in controlling soil erosion and in 42. Consider the following statements: preventing desertification. 1) The Taxus tree naturally found in the 2) It is a rich source of bio-diesel. Himalayas 3) It has nutritional value and is well-adapted 2) The Taxus tree is listed in the Red Data to live in cold areas of high altitudes. Book 4) Its timber is of great commercial value.

www.laex.in Page No. 23 https://elearn.laex.in

n

www.laex.in

Environment UPSC

Previous Year Questions

3) A drug called "Taxol" is obtained from 45. Recently, our scientists have discovered Taxus tree is effective against Parkinson's a new and distinct species of banana disease plant which attains a height of about 11 Which of the statements given above is/are metres and has orange-coloured fruit correct? pulp. In which part of India has it been a. 1 only discovered? b. 1 and 2 only a. Andaman Islands c. 2 and 3 only b. Anaimalai forests d. 1,2 and 3 c. Maikala Hills d. Tropical rain forests of northeast 43. With reference to 'Red Sanders',

sometimes seen in the news, consider 46. In a particular region in India, the local the following statements: people train the roots of living trees into 1) It is a tree species found in a part of South robust bridges across the streams. As India. the time passes, these bridges become 2) It is one of the most important trees in the stronger. These unique 'living root tropical rain forest areas of . bridges' are found in Which of the statements given above is/are a. Meghalaya correct? b. Himachal Pradesh a. 1 only c. Jharkhand b. 2 only d. Tamil Nadu

c. Both 1 and 2 47. Which one of the following groups of d. Neither 1 nor 2 plants was domesticated in the ‘New

World’ and introduced into the ‘Old 3.4 Miscellaneous

World’? 44. Consider the following statements: a. Tobacco, cocoa and rubber 1) The common Blue green algae, Spirogyra b. Tobacoo, cotton and rubber and Ulothrix are found in both fresh water c. Cotton, coffee and sugarcane ponds and oceans. d. Rubber, coffee and wheat 2) The chameleon can look ahead with one 3.5. Pesticides eye, and at the same time look behind with another. 48. In India, the use of carbofuran, methyl parathion, prorate and triazophos is Which of these statements is/are correct? viewed with apprehension. These a. 1 only Chemicals are used as b. 2 only a. Pesticides in agriculture c. Both 1 and 2 b. Preservatives in processed foods d. Neither 1 nor 2 c. Fruit-ripening agents d. Moisturizing agents in cosmetics

www.laex.in Page No. 24 https://elearn.laex.in

n

www.laex.in

Environment UPSC

Previous Year Questions

3.6. Agriculture Explanation: The Himalayan Range is very rich 49. With reference to the circumstances in in species diversity because it is a confluence of Indian agriculture, the concept of different bio-geographical zones. "Conservation Agriculture” assumes The Himalaya Hotspot is home to the world's significance. Which of the following fall highest mountains, including Mt. Everest. The under the Conservation Agriculture? mountains rise abruptly, resulting in a diversity 1. Avoiding the monoculture practices of ecosystems that range from alluvial 2. Adopting minimum tillage grasslands and subtropical broadleaf forests to 3. Avoiding the cultivation of plantation crops alpine meadows above the tree line. Vascular 4. Using crop residues to cover soil surface plants have even been recorded at more than 5. Adopting spatial and temporal crop 6,000 m. The hotspot is home to important sequencing/crop rotations populations of numerous large birds and Select the correct answer using the code mammals, including vultures, tigers, given below: elephants, rhinos and wild water buffalo. a. 1,3 and 4 Educational Objective: To know about the b. 2,3,4 and 5 biodiversity, and important places which are c. 2,4 and 5 innews. 1.1. 2 Equatorial Forest d. 1,2,3 and 5 2. Key: D 50. Consider the following agricultural Explanation: The equatorial, hot, wet climate is practices: found between 5° and 10° north and south of 1. Contour bunding the equator. Its greatest extent is found in the 2. Relay cropping lowlands of the Amazon, the Congo, Malaysia 3. Zero tillage and the East Indies. In the context of global climate change,  Equatorial rainforests are the typical which of the above helps/help in arbon vegetation type found in this region. sequestration/storage in the soil?  The region is characterized by broad-leaf a. 1 and 2 only trees with a tall and dense canopy. b. 3 only  The canopy is so dense that very little c. 1, 2 and 3 sunlight reaches the surface of equatorial d. None of them

forests. Key and Explanations Because of abundant sunlight and precipitation, the growing season exists all 1. Basics around the year. There is no distinct 1.1 Biodiversity Rich Areas drying season where the trees shed leaves. 1.1.1 Himalayan Range Shedding and blooming of leaves and fruits

1. Key: B occur throughout the year.

www.laex.in Page No. 25 https://elearn.laex.in

n

www.laex.in

Environment UPSC

Previous Year Questions

The vegetation exists in layers with the Adaptation of flora is an arbitrary option, Ethno- bottom-most layer comprising of decaying botanical importance does nothing with the organic matter, which is slow to decompose Biodiversity Hotspot selection criteria, vegetation due to low sunlight. A very dense layer of Density is also discarded. canopy exists with trees growing as tall as Educational Objective: To know about the 90 feet - this layers most of the wildlife of hotspots and its criteria to declare these forests. Above this lies the emergent 1.2 Threats to Biodiversity

layer of crowns of a few trees which 4. Key: C protrude out of the dense canopy. This Explanation: Butterflies pollinate some wild layer receives maximum sunlight and also flowers. Since some species of wasps, spiders has to endure strong winds. and birds are predators of butterflies, decline of  On the thick trunks of many of the trees, the latter could lead to a fall in the there is a layer of climbers known as the population of the predators. lianas. Epiphytes are small plants which Educational Objective: to know about the grow on the trunks and branches of these biodiversity, some of the important role played broad-leaf trees. the species.  The rainforests of Amazon are known as 5. Key: A Selvas (forests with a dense canopy) Explanation: Some of the major threats  The trees of the hardwood variety. The natural habitat of the ecosystem plays a Prominent hardwood varieties include - major role in maintaining the ecological balance. Mahogany, Ebony, dyewoods etc. Several trees are cut down every year for the  Apart from the dense hardwood forests, construction of industries, highways, settlement mangrove vegetation is also found in the and so on to fulfill the human demands. As a swamps and coastal regions. result, the species become the target to Educational Objective: To know about the predation and eventually die. natural vegetation around the world. Hunting of the wild animals for 1.1.3 Hot Spots Criteria commercialisation of their products has been a 3. Key: C major reason for the loss of biodiversity. Since Explanation: To qualify as a hotspot, a region the year 2013, more than 90 rhinos were killed must meet two strict criteria: by the poachers for their horns and as per the It must contain at least 1,500 species of records of 2016, 9 Indian Rhinos have been vascular plants (> 0.5 percent of the world’s killed in Kaziranga National Park of Assam. total) as endemics, and it has to have lost at The exploitation of the medicinal plants for least 70 percent of its original habitat. So we several laboratory purposes has resulted in the choose Species richness as well as Endemism. extinction of these species. Also, several animals Along with this Threat perception is necessary to are sacrificed for the purpose of various research take, because it makes the base of this concept. in science and medicine.

www.laex.in Page No. 26 https://elearn.laex.in

n

www.laex.in

Environment UPSC

Previous Year Questions

Natural calamities like floods, earthquakes, orangutans, gorillas and many big cats — forest fires also lead to the loss of biodiversity. are surviving on 20% or less of the land Air pollution has a major role in the loss of they once roamed. The loss of biodiversity biodiversity. Rapid cutting down of the trees has has recently accelerated. resulted in the increase of carbon dioxide in the  Several species of mammals that were atmosphere leading to climate change. As a relatively safe one or two decades ago are result, there has been an increase in the land now endangered, including cheetahs, lions and ocean temperature leaving an inimical and giraffes. There are as few as 20,000 impact on species. lions left in the wild, less than 7,000 cheetahs, 500 to 1,000 giant pandas, and about 250 Sumatran rhinoceros.  Tropical regions have seen the highest number of declining species. In South and Southeast Asia, large-bodied species of mammals have lost more than four-fifths of their historical ranges. Factors behind the loss? The main drivers of wildlife decline are habitat loss, overconsumption, pollution, invasive species, disease, as well as poaching in the case of tigers, elephants, rhinos and other large Educational Objective: To know about the animals prized for their body parts. Climate biodiversity, emerging threats to it. change is poised to become a major threat in the

6. Key: D coming decades. Explanation: The Holocene extinction, Educational Objective: To know about the otherwise referred to as the sixth mass biodiversity, emerging threats to it.and some of extinction or Anthropocene extinction, is an the terms associated with it which are in news. ongoing extinction event of species during the 7. Key: C present Holocene epoch (with the more recent Explanation: Plants that have new genes time sometimes called Anthropocene) as a result inserted into them are called transgenic or of human activity. genetically engineered / modified plants  More than 30% of animals with a backbone Prospects: — fish, birds, amphibians, reptiles and Enhanced taste and quality. mammals — are declining in both range Reduced maturation time. and population. Increased nutrients, yields, and stress tolerance.  The mammal species that were monitored Improved resistance to disease, pests, and have lost at least a third of their original herbicides. habitat. 40% of them — including rhinos,

www.laex.in Page No. 27 https://elearn.laex.in

n

www.laex.in

Environment UPSC

Previous Year Questions

Educational Objective: To learn about Educational Objective: To know about the prospects of genetically engineered plants. biodiversity, emerging threats to it and its importance. 1.3 Miscellaneous

8 Key: D 9. Key: C Explanation: Biodiversity plays a major role in Explanation: Biodiversity is higher in the low maintaining the ecological balance of the latitudes as compared to the higher latitudes as ecosystem. It refers to the number of different the habitat tolerance level is very narrow at species belonging to a particular region. In greater latitude. Mountains have high range of biodiversity, each individual species has a major species at the low latitude as they can support role to play in the ecosystem. larger number of species due to climatic Ecological Role of Biodiversity condition. Apart from providing ecological balance to the Educational Objective: to know about the environment, each individual species of biodiversity, different regions of the world, and biodiversity has a major function to play in the characteristics. ecosystem. They play a major role in the 10. Key: D production and decomposition of organic wastes, Explanation: One of the functions of The World fixing atmospheric gases and regulation of water Heritage Convention is to contain the most and nutrients throughout the ecosystem. The important and significant natural habitats for stability of the ecosystem increases with the in-situ conservation of biological diversity, diversity of the species. including those containing threatened species of Economical Role of Biodiversity outstanding universal value from the point of Biodiversity acts as a source of energy and has a view of science or conservation. Hence, major role in providing raw materials for statement 3 is correct. industrial products such as oils, lubricants, Created in 1972, the primary mission of the perfumes, dyes, paper, waxes, rubber, etc. Convention is to identify and protect the world's Importance of plant species for various natural and cultural heritage considered to be of medicinal use has been known since ages. Outstanding Universal Value. According to reports, more than 70 % of the Established in 1994, the United Nations anti-cancer drugs are derived from plants in the Convention to Combat Desertification tropical rainforests. (UNCCD) is the sole legally binding international Scientific Role of Biodiversity agreement linking environment and development Each species of the ecosystem contributes to to sustainable land management. The providing enough evidence as to how life evolved Convention addresses specifically the arid, semi- on this planet and the role of each species in arid and dry sub-humid areas, known as the maintaining the sustainability of the ecosystem. drylands The International Treaty on Plant Genetic Resources for Food and Agriculture aims at

www.laex.in Page No. 28 https://elearn.laex.in

n

www.laex.in

Environment UPSC

Previous Year Questions

o Recognizing the enormous contribution of The valley has three sub-alpine between 3,200m farmers to the diversity of crops that feed and 3,500m which is the limit for trees, lower the world. Hence, statement 1 is correct alpine between 3,500m and 3,700m, and higher o Access and benefit sharing alpine above 3,700m. Educational Objective: To know about The The rich diversity of species reflects the valley's International Treaty on Plant Genetic Resources location within a transition zone between the for Food and Agriculture, The United Nations Zanskar and Great Himalayas ranges to the Convention to Combat Desertification, The north and south, respectively, and between the World Heritage Convention etc., Eastern Himalaya and Western Himalaya flora. 2. Protected Areas Educational Objective: To know about the 2.1 National Park National parks, wildlife sanctuaries and

11. Key: B biosphere reserves classification and their Explanation: KeibulLamjao National Park, locations.

Manipur is the only floating national park in the 14. Key: D world. Its a swamp, provides refuge to Eld's deer Explanation: Namdapha National Park is the or Manipuri Deer, listed as an endangered largest protected area in the Eastern Himalaya species by IUCN. biodiversity hotspot and is located in Arunachal Educational Objective: To know about the Pradesh in Northeast India. The area is also National parks, wildlife sanctuaries and known for extensive Dipterocarp forests, biosphere reserves classification. And to know comprising the northwestern parts of the major national parks which are in news. ecoregion of Mizoram-Manipur-Kachinrain

12. Key: B forests. The habitat changes with increasing Explanation: In national parks, local people altitude from tropical moist forests to Montane arenot allowed to collect and use the forests, temperate forests and at the higher biomassavailable in the areas. elevations, to Alpine meadows and perennial Educational Objective: To know about the snow. The park has extensive bamboo forests National parks, wildlife sanctuaries and and secondary forests in addition to the biosphere reserves classification. primary forests. Educational Objective: To know about the 13. Key: D National parks, wildlife sanctuaries and Explanation: The Valley of Flowers national biosphere reserves classification and climatic park located in the Chamoli district of characteristics of the regions. Uttarakhand and is known for its meadows of endemic alpine flowers and the variety of 15. Key: D flora.Situated approximately 3255m above sea Explanation: Through Corbett National Park level, this splendid park is spread across 87.5 sq Ramganga flows (not Ganga) which is a tributary km of land. of Ganges. Through Silent Valley National Park riverBhavani flows which is a tributary of kaveri.

www.laex.in Page No. 29 https://elearn.laex.in

n

www.laex.in

Environment UPSC

Previous Year Questions

Educational Objective: To know about the Educational Objective: to know about the major national parks, wildlife sanctuaries and major national parks, wildlife sanctuaries and biosphere reserves, biosphere reserves,

16. Key: B 18. Key: D Explanation: Bhiterkanika, Orissa is a Explanation: Indravati National Park is protected area for salt water crocodile, where a national park located in Bijapur district of breeding is the main purpose of that protected Chhattisgarh, state of India. It derives its name area. from the nearby Indravati River. It is home to  Great Indian Bustard is protected in one of the last populations of rare wild buffalo. desert area of Rajasthan. Educational Objective: To know about the  The was major national parks, wildlife sanctuaries and established to protect the Niligiritahir biosphere reserves,

species. 19. Key: A Educational Objective: to know about the Explanation: Andaman and Nicobar Islands has major national parks, wildlife sanctuaries and the maximum number of National Parks. Total biosphere reserves. number of National Parks in Andaman and

17. Key: D Nicobar Islands is Nine. Explanation: Indira Gandhi Wildlife Educational Objective: to know about the Sanctuary and National Park is also known as major national parks, wildlife sanctuaries and Top Slip. It is located in Annamalai hills of Tamil biosphere reserves. Nadu. 20. Key: C Aanaimalai Tiger Reserve, earlier known Explanation: Silent Valley National Park is as Indira Gandhi Wildlife Sanctuary and situated in Nilgiri Hills of Western Ghats. The National Park (IGWLS&NP) and previously as park is bound by reserved forest to Aanaimalai Wildlife Sanctuary, is a protected the east and vested forest ofPalaghat division area located in the Tamil Nadu, and Nilamber division to the south-west India. The Tamil Nadu Environment and Forests respectively. Department by a notification dated 27 June Educational Objective: To know about the 2007, [3] declared an extent of 958.59 km2 that National parks, wildlife sanctuaries and encompassed the erstwhile IGWLS&NP or biosphere reserves classification and climatic Aanaimalai Wildlife Sanctuary, as Aanaimalai characteristics of the regions, location etc.

Tiger Reserve under the Wildlife Protection 21. Key: B Act, 1972. According to the National Tiger Explanation: In order to conserve the single Conservation Authority, the Reserve presently isolated population of Asiatic Lions, the Ministry includes a core area of 958.59 km2 and of Environment and Forests had initiated habitat buffer/peripheral area of 521.28 km2 forming a feasibility study through the Wildlife Institute of total area of 1479.87 km2. India, which inter alia has identified KunoPalpur

www.laex.in Page No. 30 https://elearn.laex.in

n

www.laex.in

Environment UPSC

Previous Year Questions

Wildlife Sanctuary inMadhya Pradesh as an Wildlife Sanctuary is a National Park, UNESCO alternate home for the Gir Lions and for the Natural World Heritage site, a Project Tiger release of wild lions from Gujarat. While State Reserve, an Elephant Reserve and a Biosphere Government of Madhya Pradesh is agreeable for Reserve in Assam. The Sundarban National Park translocation of Gir Asiatic Lions in KunoPalpur, is a National Park, Tiger Reserve, and a State Government of Gujarat has so far not Biosphere Reserve in India. It is a part of the agreed to part with Gir Asiatic Lions. Sundarbans on the Ganges Delta of India and Educational Objective: To know about Asiatic Bangladesh. Bhitarkanika National Park is a Lion and its habitat in India. national park located in the Kendrapara District 2.2 Tiger Reserve Odisha, which is not specifically for Tiger 22. Key: A reserve. Explanation: Sathyamangalam forest range is Educational Objective: To know about the a significant wildlife corridor in the Nilgiri National parks, wildlife sanctuaries and Biosphere Reserve between the Western Ghats biosphere reserves classification and climatic and the rest of the Eastern Ghats and a genetic characteristics of the regions.

link between the four other protected areas 2.3 Wildlife Sanctuaries which it adjoins, including the Billigiriranga 24. Key: A Swamy Temple Wildlife Sanctuary, Sigur Explanation: Spread over an area of 862 sq. Plateau, and km. at the foothills of the Eastern Himalayas in . Arunachal Pradesh, Pakhui Wildlife Sanctuary is also known as Pakke Wildlife Sanctuary and more popularly as Pakke Tiger Reserve. Earlier a game sanctuary, it became a wildlife sanctuary in 2001 and a part of Project Tiger in 2002. The sanctuary is well-known for being home to tigers and many amazing species of hornbill. Educational Objective: To know about the National parks, wildlife sanctuaries and biosphere reserves classification, climatic characteristics of the regions and the location.

Educational Objective: To know about the 25. Key: B major national parks and sanctuaries and their Explanation: Chilka Lake first marine strategic locations sanctuary in India, having within its bounds

23. Key: B coral reefs, mollusca, dolphins, tortoises and Explanation: Bandipur National Park, a tiger various kinds of sea birds, reserve is located in the south Indian state of Educational Objective: To know about the Karnataka. Manas National Park or Manas National parks, wildlife sanctuaries and

www.laex.in Page No. 31 https://elearn.laex.in

n

www.laex.in

Environment UPSC

Previous Year Questions

biosphere reserves classification, and their Tamil Nadu. Its flora mostly consists of tropical ecological importance. forests and is home to 2,254 species of higher

2.4 Biosphere Reserve plants including about 400 that are endemic. About 400 Red Listed Plants, 125 species of 26. Key: A orchids and rare, endemic and threatened plants Explanation: As per Forest ministry guideline have been recorded from the reserve. It is also “Biosphere Reserves are special entities (sites) home to rare endimic animals include tiger, for how human beings and nature can co-exist Asian Elephant, and NilgiriTahr. It is home to while respecting each other’s needs”. Biosphere Kanikaran tribe, one of the oldest surviving reserve- goal is to foster economic Development ancient tribes in the world. which is ecologically sustainable. transition Educational Objective: To know about the zone, the outermost part of the Biosphere National parks, wildlife sanctuaries and Reserve, is an area of active biosphere reserves classification, climatic cooperation between the management and characteristics of the regions and the location. the local people, wherein activities like 28. Key: B settlements, cropping, forestry , recreation Explanation: UNESCO has introduced the and other economic that are in harmony with category of “Biosphere Reserve” to be nominated the conservation goals by the national government and remains under Educational Objective: To know about the the sovereign jurisdiction of the state in which National parks, wildlife sanctuaries and they are located. They are internationally biosphere reserves classification, climatic recognised. 11 of the 18 biosphere reserves in characteristics of the regions and the location.

India are a part of the World Network of 27. Key: A Biosphere Reserves. Explanation: ABR in situated at the southern- Nanda Devi Biosphere Reserve - Uttarakhand, most end of the Western Ghats and spread over Nokrek Biosphere Reserve - Meghalaya, two southern states Kerala and Tamil Nadu. It Pachmarhi Biosphere Reserve -Madhya Pradesh, was established in 2001. It is named after Simlipal Biosphere Reserve - Odisha, Great Mala peak that rises up to almost 1868 Nicobar Biosphere Reserve - Great Nicobar, metres above sea level, in , Achanakmar-Amarkantak Biosphere Reserve - Kerala. In March 2016, it was included in the Chhattisgarh, Madhya Pradesh, Agasthyamalai World Network of Biosphere Reserves of Biosphere Reserve- Kerala and Tamil Nadu. UNESCO. Kanchenjunga Biosphere Reserve added in ABR covers an area of 3,500 sq km at an 2018 (this question was asked earlier before.) altitude ranging from 100 metres to 1,868 Educational Objective: To know about the metres above the Mean Sea Level. It covers National parks, wildlife sanctuaries and Peppara and Shendurney wildlife sanctuaries biosphere reserves classification, climatic and parts of the Neyyar sanctuary in Kerala and characteristics of the regions and the location. the KalakadMundanthurai Tiger Reserve of World Network of Biosphere Reserves

www.laex.in Page No. 32 https://elearn.laex.in

n

www.laex.in

Environment UPSC

Previous Year Questions

29. Key: D Educational Objective: To know about the Explanation: Similipal biosphere reserve is in National parks, wildlife sanctuaries and Orissa. DehongDeband biosphere reserve is in biosphere reserves classification, climatic Arunachal Pradesh, Nokrek biosphere reserve is characteristics of the regions and the in Meghalaya and Kanchenjunga biosphere location.World Network of Biosphere Reserves. reserve is situated in Sikkim. 2.5 Eco-Sensitive Zones

Educational Objective:To know about the 31. Key: D National parks, wildlife sanctuaries and Explanation: Eco-Sensitive Zones are the areas biosphere reserves classification, climatic that are declared under the Environment characteristics of the regions and the location. (Protection) Act, 1986. Tourism and other

30. Key: B activities allowed in these areas. Explanation: Biosphere reserves are areas of Educational Objective: To know about the terrestrial and coastal or marine ecosystems or major eco sensitive zones, statutory provisions.

its amalgamation. The biosphere reserve 2.6 Mix of above Categories network was launched in 1971 by UNESCO, two 32. Key: C years after the initiation of MAB- Man and the Explanation: Dampa Tiger Reserve, the largest biosphere programme. Government of India wildlife sanctuary in Mizoram. Saramati peak established 18 biospheres in the country. is in Nagaland. It is located near Tuensang town Name of Biosphere Reserve with a height of 3,826 m. Gumti Wildlife Nilgiri Nanda Devi Sanctuary is famous wildlife reserve in Tripura. Nokrek Educational Objective: to know about the Great Nicobar National parks, wildlife sanctuaries and Gulf of Mannar biosphere reserves classification, climatic Manas characteristics of the regions and the location. Sunderbans 33. Key: C Simlipal Explanation: Nokrek is the highest peak in Dibru-Saikhowa Garo Hills. Dehang-Dibang Loktaklake and Barailrange are away from Pachmarhi eachother. Khangchendzonga Educational Objective: to know about the Agasthyamalai National parks, wildlife sanctuaries and Achanakamar – Amarkantak biosphere reserves classification, climatic

Kachchh characteristics of the regions and the Cold Desert location.World Network of Biosphere Reserves

Seshachalam Hills 34. Key: C Panna

www.laex.in Page No. 33 https://elearn.laex.in

n

www.laex.in

Environment UPSC

Previous Year Questions

Explanation: A biosphere reserve is a unique Nagerhole National Park - Karnataka concept, which includes one, or more protected Kugti Wildlife Sanctuary - Himachal areas and surrounding lands that are managed Pradesh to combine both conservation and sustainable Sultanpur Bird Sanctuary - Haryana use of natural resources. In India 13 states in Educational Objective: to know about the have Biosphere Reserves and the funds National parks, wildlife sanctuaries and allocated to them is utilized to monitor and biosphere reserves classification, climatic promote bio-diversity conservation through characteristics of the regions and the location. surveys and assessment of flora and fauna. World Network of Biosphere Reserves

Basically Biosphere reserves are to protect larger 37. Key: B areas of natural habitat, and often include one Explanation: or more national parks and/or preserves, along Bondla Wildlife Sanctuary Goa buffer zones that are open to some economic Kangerghat National Park Chhattisgarh uses as per the IUCN Category V. Orang Sanctuary Assam The difference between a national park and a Ushakothi Wildlife Sanctuary Orissa sanctuary is that no human activity is allowed Educational Objective: To know about the inside a national park, while limited activities National parks, wildlife sanctuaries and are permitted within the sanctuary. biosphere reserves classification, climatic Educational Objective:To know about the characteristics of the regions and the location. National parks, wildlife sanctuaries and World Network of Biosphere Reserves. biosphere reserves classification, climatic 3. Plant Diversity characteristics of the regions and the 3.1 Use/Importance of Few Species location.World Network of Biosphere Reserves

38. Key: D 35. Key: C Explanation: In many parts of the world, Explanation: natural resources are the only source of Chandra Prabha - Varanasi livelihood opportunities available to people. Kerera - Jhansi Girardiniadiversifolia (Himalayan nettle), a Jaisamand - Udaipu fibre-yielding plant, has become an important Nahargarh - Jaipur livelihood option for people living in the remote Educational Objective: to know about the mountainous villages of the Hindu Kush National parks, wildlife sanctuaries and Himalaya. There is a community in Khar, a biosphere reserves classification, climatic hamlet in Darchula district in far-western Nepal, characteristics of the regions and the which produces fabrics from Himalayan nettle. location.World Network of Biosphere Reserves The fabric and the things made from it are sold

36. Key: B in local as well as national and international Explanation: markets as high-end products. KangerGhati National Park - Chhattisgarh

www.laex.in Page No. 34 https://elearn.laex.in

n

www.laex.in

Environment UPSC

Previous Year Questions

Educational Objective: To know about the Educational Objective: to know about the importance of flora and fauna. importance of flora and fauna.

39. Key: C 3.3 General information of Individual Explanation: Buckthorn Species It has multi-purpose medicinal and nutritional 42. Key: B properties, and also helps in soil conservation Explanation: Taxuswallichiana, the Himalayan and nitrogen fixation. yew, is a species of yew, native to the Himalaya Hardy, drought-resistant and tolerant to extreme and parts of south-east Asia. The species has a temperatures from – 43º C to + 40º C, the plant variety of uses in traditional medicine. It is has an extensive root system which can fix currently classified as endangered by the IUCN. atmospheric nitrogen, making it ideal for Educational Objective: to know about the controlling soil erosion and preventing importance of flora and fauna. Important flora desertification. which are in the news. National mission on buckthorn in is a part of 43. Key: A Sub-Mission on Cold Desert Ecosystems under Explanation: Red Sanders (a tree) has a highly the Green India Mission — which is a part of the restrictive distribution in the South Eastern National Action Plan on Climate Change. portion of Indian peninsula (Eastern Ghats of Educational Objective: To know about the South India) to which it is endemic. It occurs in importance of flora and fauna.

the forest formation which is classified as 3.2 Insectivorous “Southern Tropical Dry Deciduous Forests” t 40. Key: B is generally found at altitudes of 150 – 900 m. It Explanation: Insectivorous plants prey insects grows on dry, hilly, often rocky ground, and because most of the insectivorous plants grow in occasionally found on precipitous hill sides also. those area where there is deciet of nitrogen in It prefers lateritic and gravelly soil and cannot nutrients so they catch insect to full its need of tolerate water logging. nutrients. Educational objective: to know about the Educational Objective:To know about the importance of flora and fauna. importance of flora and fauna. 3.4 Miscellaneous

41. Key: B 44. Key: B Explanation: Pitcher plants are several Explanation: SpirogyraandUlothrix are green different carnivorous plants which have modified algae not blue algae. The chamelean can see in leaves known as pitfall traps—a prey-trapping both the direction forward and backward at the mechanism featuring a deep cavity filled with same time. digestive liquid. The traps of what are Educational Objective: To know about the considered to be "true" pitcher plants are formed ecological niche of species, importance to by specialized leaves. The plants attract and human. drown their prey with nectar.

www.laex.in Page No. 35 https://elearn.laex.in

n

www.laex.in

Environment UPSC

Previous Year Questions

45. Key: A Educational Objective: to know about the Explanation: Scientists at the Botanical Survey natural vegetation and its importance on human of India (BSI) have discovered a new species of life.

banana from a remote tropical Krishna Nalah 47. Key: A rain forest on the Little Andaman islands. The Explanation: Tobacco, cocoa and rubber were new species is about 11 metres high, whereas as originally domesticated or cultivated in the ‘New the usual banana species is about three to four World’ (America) and introduced into the ‘Old meters high. The fruit pulp is orange in colour, World’ (Asia and Africa). Cotton and Wheat are distinctive from the white and yellow color of being cultivated in India since very ancient regular bananas. Hence (a) is the correct times. People of Mehrgarh (Baluchistan, answer. Pakistan) cultivated Cotton during Neolithic age. Educational Objective: To know about the Wheat was cultivated by people of Harappan newly discovered species and locations civilisation and Vedic Aryans associated with it. Educational Objective: To know about the

46. Key: A different vegetation. Explanation: Root Bridge 3.5 Pesticide Living root bridges (also known as Jing KiengJri) 48. Key: A are the aerial bridges that are built by weaving Explanation: The Ministry of Agriculture has and manipulating the roots of the Indian rubber issued a notification dated December 15, 2016 tree. inviting objections and suggestions from the  A root bridge uses traditional tribal stakeholders on completely banning of 12 knowledge to train the roots of the Indian pesticides/insecticides and phasing out of 6 rubber tree (found in abundance in the pesticides/insecticides. area) to grow laterally across a stream bed These pesticides include: Benomyl, Carbaryl, resulting in a living bridge of roots. Diazinon, Fenarimol, Fenthion, Linuron, MEMC,  It spans between 15 and 250 feet and is Methyl Parthion, Sodium Cyanide, Thiometon, built over centuries. Tridemorph, Trifluralin, Alachlor, Dichlorvos,  They have been serving as connectors for Phorate, Phosphamidon, Triazophos and generations in the Indian state Trichlorfon. Hence, statement 1 is correct. of Meghalaya. Educational Objective: To know chemicals  The bridges are primarily a means to cross used as pesticides in agriculture. streams and rivers. They have also become 3.6 Agriculture world-famous tourist attractions. The two 49. Key: B most popular tourist spots are- Riwai Root Explanation: Conservation Agriculture (CA) Bridge and Umshiang Double Decker aims to achieve sustainable and profitable Bridge. agriculture and improve farmers’ livelihoods. CA observes three main principles

www.laex.in Page No. 36 https://elearn.laex.in

n

www.laex.in

Environment UPSC

Previous Year Questions

 Minimal soil disturbance – (i.e. no tillage) in the desired position to germinate. Hence, through direct seed and/or fertilizer placement. statement 3 is correct.  A permanent soil cover - (at least 30 Educational Objective: To know about methods percent) with crop residues and/or cover crops. that help in carbon sequestration in soil.  rotation of crops / Species diversification - through varied crop sequences and associations IV Animal Biodiversity involving at least three different crops 1. Distribution and Habitat Related

1. Consider the following statements: 1) Asiatic lion is naturally found in India only. 2) Double-humped camel is naturally found in India only. 3) One-horned rhinoceros is naturally found 50. Key: C in India only. Explanation: Carbon sequestration is the Which of the following statement given above process of capture and long-term storage of is/are correct? atmospheric carbon dioxide to mitigate global a. 1 only warming and to avoid dangerous impacts of b. 2 only climate change. c. 1 and 3 only Soil is one of the largest reservoirs, where d. 1,2 and 3 carbon could be restored. So, methods of soil 2. Consider the following pairs conservation help in carbon sequestration. Wildlife Naturally found in Contour bunding involves the construction of 1) Blue- finned Mahseer : Cauvery River banks along the contours. Terracing and 2) Irrawaddy dolphin : Chambal River contour bunding which divide the hill slope into 3) Rusty-spotted cat : Eastern Ghats numerous small slopes, check the flow of water, Which of the pairs given above are correctly promote absorption of water by soil and save soil matched? from erosion. Hence, statement 1 is correct a. 1 and 2 only Relay cropping: Relay cropping involves b. 2 and 3 only growing of two or more crops on the same field c. 1 and 3 only with the planting of the second crop after the d. 1,2 and 3 first one has reached its reproductive stage. It 3. If you want to see gharials in their saves soil from erosion. Hence, statement 2 is natural habitat, which one of the correct following is the best place to visit? Zero Tillage farming is the method of farming a. Bhitarkanika Mangroves in which there is no tilling involved. Here, the b. Chambal River soil is left undisturbed except to place the seeds c. Pulicat Lake d. DeeporBeel

www.laex.in Page No. 37 https://elearn.laex.in

n

www.laex.in

Environment UPSC

Previous Year Questions

4. In which of the following regions of India 2) Monitor lizard are you most Likely to come across the 3) Pygmy hog 'Great Indian Hornbill' in its natural 4) Spider monkey habitat? Which of the above are naturally found in a. Sand deserts of northwest India India? a. 1, 2 and 3 only b. Higher Himalayas of Jammu and Kashmir c. Salt marshes of western Gujarat b. 2 and 3 only d. Western Ghats c. 1 and 4 only d. 1,2,3 and 4 5. If you walk through countryside, you are

likely to see some birds stalking alongside 8. Consider the following: the cattle to seize the insects disturbed by 1) Black-necked crane their movement through grasses. Which of 2) Cheetah the following is/are such bird/birds? 3) Flying squirrel 1) Painted Stork 4) Snow leopard 2) Common Myna 3) Black-necked Crane Which of the above are naturally found in Select the correct answer using the code given India? below. a. 1, 2 and 3 only a. 1 and 2 b. 1, 3 and 4 only b. 2 only c. 2 and 4 only c. 2 and 3 d. 1, 2, 3 and 4 d. 3 only 2. General Information/Unique 6. In which of the following States is lion- Feature of Some Species tailed macaque found in its natural 2.1 King Cobra habitat? 1) Tamil Nadu 9. King Cobra is the only snake that makes 2) Kerala its own nest. Why does it make its nest? 3) Karnataka a. It is a snake-eater and the nest helps 4) Andhra Pradesh attract other snakes

Select the correct answer using the codes b. It is a viviparous snake and needs a nest to give birth to its offspring given below c. It is an oviparous snake and lays its eggs a. 1,2 and 3 only in the nest and guards the nest until they b. 2 only are hatched c. 1,3 and 4 only d. It is a large, cold blooded animal and needs d. 1,2,3 and 4 a nest to hibernate in the cold season

7. Consider the following: 1) Star tortoise

www.laex.in Page No. 38 https://elearn.laex.in

n

www.laex.in

Environment UPSC

Previous Year Questions

10. For which one of the following snakes is 2.4 GangeticDolphin

the diet mainly composed of other 13. Which one of the following is the snakes? national aquatic animal of India? a. Krait a. Saltwater crocodile b. Russell’s viper b. Olive ridley turtle c. Rattlesnake c. Gangetic dolphin d. King cobra d. Gharial

2.2 Kharai Camel 14. Other than poaching, what are the

11. What is/are unique about 'Kharai camel' possible reasons for the decline in the a breed found in India? population of Ganges River Dolphins? 1) It is capable of swimming up to three 1) Construction of dams and barrages on kilometers in seawater. rivers 2) It survives by grazing on mangroves. 2) Increase in the population of crocodiles in 3) It lives in the wild and cannot be rivers domesticated. 3) Getting trapped in fishing nets accidentally Select the correct answer using the code 4) Use of synthetic fertilizers and other given below. agricultural chemicals in crop-fields in the a. 1 and 2 only vicinity of rivers. b. 3 only Select the correct answer using the code c. 1 and 3 only given below. d. 1, 2 and 3 a. 1 and 2 only

2.3 Dugong b. 2 and 3 only

12. With reference to 'dugong', a mammal c. 1, 3 and 4 only found in India which of the following d. 1, 2, 3 and 4

statements is/is correct? 2.5 Wild Ass 1) It is a herbivorous marine animal. 15. A sandy and saline area is the natural 2) It is found along the entire coast of India. habitat of an Indian animal species. The 3) It is given legal protection under Schedule I animal has no predators in that area but of the Wildlife (Protection) Act; 1 972. its existence is threatened due to the Select the correct answer using the code destruction of its habitat. given below. Which one of the following could be that a. 1 and 2 animal? b. 2 only a. Indian wild buffalo c. 1 and 3 b. Indian wild ass d. 3 only c. Indian wild boar d. Indian gazelle

www.laex.in Page No. 39 https://elearn.laex.in

n

www.laex.in

Environment UPSC

Previous Year Questions

d. Orangutan 2.6 Bat

16. In the context of Indian wild life, the 21. Consider the following statements: flying fox is a 1) Toothless mammals such as pangolins are a. Bat not found in India b. Kite 2) Gibbon is the only ape found in India c. Stork Which of the statements given above is/are d. Vulture correct? a. 1 only 2.7 Panda b. 2 only 17. The Panda belongs to the same family as c. Both 1 and 2 that of d. Neither 1 nor 2 a. Bear 2.11 Gambusia Fish b. Cat c. Dog 22. The release of which one of the d. Rabbit following into ponds and wells helps in 2.8 Spider controlling the mosquitoes? a. Crab 18. In which one of the following kinds of b. Dogfish organisms is the phenomenon found c. Gambusia fish wherein the female kills the male after d. Snail. copulation? 2.12 Octopus a. Dragonfly 23. Octopus is b. Honeybee a. An arthropod c. Spider b. An echinoderm d. Pit viper c. A hemichordate 2.9 Echidna d. A mollusc 19. Among the following, Which one lays 2.13 Vulture eggs and does not produce young one 24. Vultures which used to be very common in directly? Indian countryside some years ago are a. Echidna rarely seen nowadays. This is attributed to b. Kangroo a. The destruction of their nesting sites by new c. Porcupine invasive species d. Whale b. A drug used by cattle owners for treating 2.10 Ape Species their diseased cattle

20. Among the following, which one is not c. Scarcity of food available to them an ape? d. A widespread, persistent and fatal disease a. Gibbon among them b. Gorilla c. Langur

www.laex.in Page No. 40 https://elearn.laex.in

n

www.laex.in

Environment UPSC

Previous Year Questions

2.14 Echinoderms 3.2 Marsupials

25. Which one of the following statement is 28. A class of animals known as Marsupials not correct? is a characteristic feature of a. All echinoderms are viviparous a. Africa b. Roundworm has no circulatory system b. Australia c. In bony fishes, swim bladder is usually c. South America present d. South-east Asia. d. In cartilaginous fishes, fertilization is 3.3 Mix internal 29. Consider the following statements: 2.15 Amoeba 1) Tapeworm is a hermaphrodite 26. A: Amoeba reproduces by fission. 2) Round-worm has separate sexes R: All unicellular organisms reproduce 3) Filaria is caused by a nematode by asexual methods. 4) Guinea-worm is an annelid a. Both A are R are true and R is the correct Which of these are correct? explanation of A a. 1 and 2 b. Both A and R are true but R is not a b. 1,2 and 3 correct explanation of A c. 3 and 4 c. A is true but R is false d. 2,3 and 4 d. A is false but R is true 3.4 Keystone Species 3. Miscellaneous 30. Within biological communities, some 3.1 Scientific Names species are important in determining 27. Match List-I with List-II and select the the ability of a large number of other correct answer: species to persist in the community. List-I List-II Such species are called (Indian wildlife species) : (Scientific name) a. Keystone species A. Asiatic Wild Ass : Boselaphus b. Allopatric species tragocamelurs c. Sympatric species B. Barasingha : Cervusduvauceli d. Threatened species

C. Chinkara : Equushemionus 3.5 Insects D. Nilgai : GazellaGazella 31. How do most insects respire? Codes: a. Through skin a. A-2; B-3; C-1; D-4 b. Through gills b. A-3; B-2; C-4; D-1 c. By lungs c. A-2; B-3; C-4; D-1 d. By tracheal system d. A-3; B-2; C-1; D-4 3.6 Mammals

32. Consider the following Animals

www.laex.in Page No. 41 https://elearn.laex.in

n

www.laex.in

Environment UPSC

Previous Year Questions

1) Sea cow 3) CITES is legally binding on the States that 2) Sea horse have joined it, but this Convention does 3) Sea lion not take the place of national laws. Which of the above is / are mammal Select the correct answer using the code /mammals? given below. a. 1 only a. 1 only b. 1 and 3 only b. 2 and 3 only c. 2 and 3 only c. 1 and 3 only d. 1,2 and 3 d. 1,2 and 3 only

3.7 Pollinating Agents 35. The "Red Data Books" published by the

33. Consider the following kinds of International Union for Conservation of organisms Nature and Natural Resources (IUCN) 1) Bat contain lists of 2) Bee 1) Endemic plant and animal species present 3) Bird in the biodiversity hotspots. 2) Threatened plant and animal species. Which of the above is/are pollinating agent / 3) Protected sites for conservation of nature agents? and natural resources in various countries. a. 1 and 2 only Select the correct answer using the codes b. 2 only given below: c. 1 and 3 only d. 1, 2 and 3 a. 1 and 3 b. 2 only 4. Conservation Status and c. 2 and 3 Conservation Efforts d. 3 only 4.1 IUCN 34. With reference to the International 4.2 Endangered Species

Union for Conservation of Nature and 36. Which one of the following groups of Natural Resources(IUCN) and the animals belongs to the category of Convention on International Trade in endangered species? Endangered Species of Wild Fauna and a. Great Indian Bustard, Musk Deer, Red Flora (Cities), which of the following Panda and Asiatic Wild Ass statements is/are correct? b. Kashmir Stag, Cheetal, Blue Bull and 1) IUCN is an organ of the United Nations and Great Indian Bustard CITES is an international agreement c. Snow Leopard, Swamp Deer, Rhesus between governments Monkey and Saras (Crane) 2) IUCN runs thousands of field projects d. Lion-tailed Macaque, Blue Bull, Hanuman around the world to better manage natural Langur and Cheetal. environments.

www.laex.in Page No. 42 https://elearn.laex.in

n

www.laex.in

Environment UPSC

Previous Year Questions

37. Consider the following fauna of India: d. The World Sustainable Development 1. Gharial Summit 2016, New Delhi

2. Leatherback turtle 41. With reference to 'Agenda 21', 3. Swamp deer sometimes seen in the news, consider Which of the above is/are endangered? the following statements: a. 1 and 2 only 1. It is a global action plan for sustainable b. 3 only development. c. 1,2 and 3 2. It originated in the World Summit on d. None of the above Sustainable Development held in 4.3 In-situ Method Johannesburg in 2002. 38. Which one of the following is not a site Which of the statements given above is / are for in-situ method of conservation of correct? flora? a. 1 only a. Biosphere Reserves b. 2 only b. Botanical Garden c. Both 1 and 2 c. National Park d. Neither 1 nor 2

d. Wildlife Sanctuary 42. Consider the following statements: 4.4 M- Stripe 1. The Sustainable Development Goals were 39. The term 'M-STRIPES' is sometimes first proposed in 1972 by a global think seen in the news in the context of tank called the Club of Rome'. a. Captive breeding of Wild Fauna 2. The Sustainable Development Goals have b. Maintenance of Tiger Reserves to be achieved by 2030. c. Indigenous Satellite Navigation System Which of the statements given above is / are d. Security of National Highways. correct?

5. Sustainable Development a. 1 Only b. 2 only 40. The Partnership for Action on Green c. Both 1 and 2 only Economy (PAGE),a UN mechanism to d. Neither 1 nor 2 assist countries transition towards 43. Consider the following statements greener and more inclusive economies, regarding 'Earth Hour': emerged at 1. It is an initiative of UNEP and UNESCO. a. The Earth Summit on Sustainable 2. It is a movement in which the participants Development 2002, Johannesburg switch off the lights for one hour on a b. The United Nations Conference on certain day every year. Sustainable Development 2012, Rio de 3. It is a movement to raise the awareness Janeiro about the climate change and the need to c. The United Nations Framework Convention save the planet. on Climate Change 2015, Paris

www.laex.in Page No. 43 https://elearn.laex.in

n

www.laex.in

Environment UPSC

Previous Year Questions

Which of the statements given above is/ are a. 1 and 2 only correct b. 3 only a. 1 and 3 only c. 2 and 3 only b. 2 only d. 1, 2 and 3

c. 2 and 3 only 46. With reference to an organization known d. 1, 2 and 3 as 'Birdlife International', which of the

6. Bio- Diversity Global Efforts following statements is/are correct?

44. Consider the following statements in 1) It is a Global Partnership of Conservation respect of Trade Related Analysis of Organizations. Fauna and Flora in Commerce 2) The concept of 'biodiversity hotspots' (TRAFFIC): originated from this organization. 1. TRAFFIC is a bureau under United Nations 3) It identifies the sites known/ referred to as - Environment Programme (UNEP) 'Important Bird and Biodiversity Areas'. 2. The mission of TRAFFIC is to ensure that Select the correct answer using the code trade in wild plants and animals is not a given below. threat to the conservation of nature. a. 1 only Which of the above statements is/are b. 2 and 3 only correct? c. 1 and 3 only a. 1 only d. 1, 2 and 3 b. 2 only 7. Miscellaneous c. Both 1 and 2 47. Consider the following statements: d. Neither 1 nor 2 1. Salt-water crocodile is found in the

45. With reference to an initiative called Andaman and Nicobar Islands. 'The Economics of Ecosystems and 2. Shrew and tapir are found in the Western Biodiversity (TEEB)" which of the Ghats of the Malabar region. following statements is/are correct? Which of the statements given above is/are 1. It is an initiative hosted by UNEP, IMF and correct? World Economic Forum. a. 1 only 2. It is a global initiative that focuses on b. 2 only drawing attention to the economic benefits c. Both 1 & 2 of biodiversity. d. Neither 1 nor 2

3. It presents an approach that can help 48. Match List-I with List-II and select the decision-makers recognize, demonstrate correct answer usingthe codes given and capture the value of ecosystems and below the lists: biodiversity List-I (Bone) List-II (Name) Select the correct answer using the code A. Breast bone 1. Clavicle given below, B. Collar bone 2. Patella

www.laex.in Page No. 44 https://elearn.laex.in

n

www.laex.in

Environment UPSC

Previous Year Questions

C. Knee-cap 3. Scapula India: Girwa River, Chambal River, Ken River, D. Shoulder blade 4. Sternum Son River, Mahanadi River, Ramganga River Codes: Nepal:Rapti-Narayani River a. A-4; B-1; C-3; D-2 Educational Objective:To know about the b. A-1; B-4; C-3; D-2 Distribution of Habitats.

c. A-1; B-4; C-2; D-3 4. Key: D d. A-4; B-1; C-2; D-3 Explanation: Great Hornbill or Great Indian

Hornbill is one of the larger member of the Key and Explanations hornbill family. It is found in South and South-

1. Distribution and Habitat Related East Asia. They are also found in Western

1. Key: A Ghats. Under IUCN Red List, great Indian Explanation: Asiatic lions are slightly smaller Hornbill comes under Near Threatened. than African lions. At present Gir National Park Educational Objective: to know about the and Wildlife Sanctuary is the only natural abode Distribution of Habitats and classification.

of the Asiatic lion. Double-hump camels are 5. Key: B found naturally in Tibet, Mountain regions of Explanation: Common Mynaare birds stalking China, Mangolia, Pakistan and also in mountain alongside the cattle to seize the insects regions of Afghanistan. One horned rhino is disturbed by their movement through grasses. found in India, Pakistan,Nepal and lower The common myna is readily identified by the reached of Bhutan. brown body, black hooded head and the bare Educational Objective: to know about the yellow patch behind the eye. Distribution of Habitat in different regions Educational Objective: To know about the around the world Distribution of Habitats, different ecological

2. Key: C regions associated with the habitats

Explanation: 6. Key: A Blue- finned Mahseer : Cauvery River Explanation: lion-tailed macaque Irrawaddy dolphin : Chilkalake Mainlydiurnalarboreal, it prefers the upper Rusty-spotted cat : Eastern Ghats canopy of primary tropical evergreen rainforest Educational Objective: To know about the but may also be found in monsoon forest in hilly Distribution of Habitats country and in disturbed forest.

3. Key: B It can be found in Karnataka, Tamil Nadu and Explanation: Gharialsonce inhabited all the Kerala in the western ghat region. major river systems of the Indian Subcontinent, Unlike other macaques, it avoids humans from the Irrawaddy Riverin the east to the Indus Educational Objective: to know about the River in the west. Their distribution is now Distribution of Habitats.

limited to only 2% of their former range 7. Key: A

www.laex.in Page No. 45 https://elearn.laex.in

n

www.laex.in

Environment UPSC

Previous Year Questions

Explanation: Star tortoise is found in India in viper and hump-nosed pit viper by following the dry and scrub forests. Pygmy Hog is an their odour trails. endangered species found in Assam. Only 150 Educational Objective: to know about the animals are left. Monitor Lizard is found in important species, and their unique features. India, Sri Lanka and Pakistan. Spider Monkey 2.2 Kharai Camel is the inhabitant of tropical forests of Central 11. Key: A and South America. Explanation: Kharai camels have a special Educational Objective: to know about the ability to swim in seawater and feed on saline Distribution of Habitats. plants and mangroves, which is how they get

8. Key: B their name, Kharai ('salty' in Gujarati). They are Explanation: Species naturally found in India also known as dariyataru (meaning sea- 1) Black-necked crane swimmer). Due to their unique diet, 2) Flying squirrel these camels swim to islands near the shore in 3) Snow leopard search of food. Educational Objective: to know about the Educational Objective: To know about the Distribution of Habitats. important species, and their unique features. 2.3 Dugong 2. General Information/Unique Feature 12. Key: C of Some Species Explanation: The dugong is a medium- 2.1 King cobra sized marine mammal. It is one of four living 9. Key: B species of the order Sirenia, which also includes Explanation: The king cobra is the only snake three species of manatees. It is the only living that builds a nest using dry leave litter, starting representative of the once-diverse from late March to late May. Most nests are family Dugongidae; its closest modern located at the base of trees, are up to 55 cm relative, Steller's sea cow (Hydrodamalisgigas), (22 in) high in the center and 140 cm (55 in) was hunted to extinction in the 18th century. wide at the base. They consist of several layers The dugong is the only and have mostly one chamber, into which the strictly herbivorous marine mammal. female lays eggs. Educational Objective: To know about the Educational Objective: To know about the important species, and their unique features. important species, and their unique features.

2.4 Gangetic Dolphin

10. Key: D 13. Key: C Explanation: The king cobra's diet consists Explanation: The Gangetic dolphins have been primarily of other snakes and lizards, declared as the National Aquatic Animal of India including Indian cobra, banded krait, rat .River Dolphin is the National Aquatic Animal of snake, pythons, green whip snake, keelback, India. The Ministry of Environment and banded wolf snake and Blyth's reticulated Forests notified the Ganges River Dolphin as snake It also hunts Malabar pit the National Aquatic Animal on 18th May

www.laex.in Page No. 46 https://elearn.laex.in

n

www.laex.in

Environment UPSC

Previous Year Questions

2010. This mammal is also said to represent the individuals in and outside of the Wild Ass purity of the holy Ganga as it can only survive in Wildlife Sanctuary of India. pure and fresh water. Educational Objective:To know about the Educational Objective: to know about the important species, and their unique features. important species, and their unique features. And species classification by international organizations and Acts. 14. Key: C 2.6 Bat Explanation: Ganges River Dolphin is placed under “Endangered Category” in the IUCN 16. Key: A Red List. It lives in one of the world’s most Explanation: The Indian flying fox also known densely populated areas, and is threatened by as the greater Indian fruit bat, is a species removal of river water and siltation arising from of flying fox found in South Asia. It is one of the deforestation, pollution and entanglement in largest bats in the world. It is of interest as sheries nets. They have been poached over for a disease vector, as it is capable of transmitting their oil. The habitat degradation due to several viruses to humans. It is nocturnal and declining ow, heavy siltation and construction of feeds mainly on ripe fruits, such barrages causing physical barrier for this as mangoes and bananas, and nectar. This migratory species is also one of the reasons species is often regarded as vermin due to its behind decline of their numbers. destructive tendencies towards fruit farms, but Educational Objective: To know about the the benefits of its pollination and seed important species, and their unique features. propagation often outweigh the impacts of its And species classification by international fruit consumption. organizations and Acts. Educational Objective: to know about the important species, and their unique features. 2.5 Wild Ass 2.7 Panda 15. Key: B Explanation: The Indian 17. Key: A ass(Equushemionuskhur)also called Explanation: Pandas descend from the family theGhudkhur,KhurorIndian onager in the local of Bears. Gujarati language, is a subspecies of Educational Objective:To know about the the onager native to Southern Asia. important species, and their unique features. 2.8 Spider It is currently listed as Near Threatenedby IUCN. At the previous census in 18. Key: C 2009, estimated 4,038 Indian wild ass. However, Explnation:Sexual cannibalism is identified only the population was still growing. In December in spiders. In which female organism kills the 2014, the population was estimated at 4,451 male before during individuals. It has increased from a jump of 454. or after copulation. In most species in which it However, as of 2015, the current Indian wild ass occurs, sexual connibalism is related to the population has increased to more than 4,800 larger size of female due to sexual dimorphism.

www.laex.in Page No. 47 https://elearn.laex.in

n

www.laex.in

Environment UPSC

Previous Year Questions

Educational Objective: To know about the Explanation: Octopus is an animal of class- important species, and their unique features. Cephalopoda and phylum Mollusca. The shell is 2.9 Echidna absent. It is found at bottom of the sea. It kills

its prey with poisonous saliva. It can change its 19. Key: A colour. Explanation: Echidnas sometimes known as Educational Objective:To know about the spiny anteaters, belong to the family important species, and their unique features. Tachyglossidae in the monotreme order of egg- laying mammals. The four extant species 2.13 Vulture of Echidnas and the platypus are the only living 24. Key: B mammals that lay eggs and the only surviving Explanation: The major reason behind the members of the order Monotremata. vulture population getting nearly wiped out was Educational Objective:To know about the the drug Diclofenac. It was found in the carcass important species, and their unique features. of cattle on which the vultures feed. 2.10 Ape Species  The drug was commonly administered to 20. Key: C cattle to treat inflammation. Explanation: Gibbon, Gorilla and Orangutan  Its veterinary use was banned in 2008 by are apes, but Langur is not an ape, it is the Government of India. prosimians.  Bioaccumulation (the gradual Educational Objective: to know about the accumulation of substances, such as important species, and their unique features. pesticides, or other chemicals in an 21. Key: B organism) of Diclofenac caused kidney Explanation: Pangolins is found in India, Sri failure in Vultures, leading to death. Lanka, Nepal and some parts of Pakistan.  Diclofenac is dangerously fatal for Gibbons are the only apes found in India. The Vultures. Even 1% of it in carcass would Hoollongapar Gibbon Sanctuary is in Assam kill the Vulture in a short time after it which is a safe recluse for gibbons. feeds such carcass. Educational Objective: To know about the  The poisoned carcasses were dumped to important species, and their unique features. kill some local stray animals. But when 2.11 Gambusia Fish vultures fed on them, it became one of the 22. Key: C vital reasons leading to their death. Explanation: Gambusiaaffinis is anlarvivorous  It is imperative to manage our carcass fish. This fish is released into ponds and wells to dumps and make sure that poisoned controlling the mosquitoes by eating mosquito carcasses are not dumped for the vultures larvae, insects and crustaceans. to feed on. Educational Objective: To know about the  The forest department cremates the animal important species, and their unique features. carcasses instead of burying them, to keep 2.12 Octopus the poachers away. But this practice is 23. Key: D

www.laex.in Page No. 48 https://elearn.laex.in

n

www.laex.in

Environment UPSC

Previous Year Questions

denying food to vultures leading to their deer species distributed in the Indian death out of starvation. subcontinent. Educational Objective: To know about the The chinkara (Gazellabennettii), also known as important species, and their unique features the Indian gazelle, is a gazelle species native to 2.14 Echinoderms Iran, Afghanistan, Pakistan and India.

25. Key: A Educational Objective: to know about the Explanation: The reproduction takes place by important species, and their unique features.

Asexual and Sexual methods. Examples of 3.2 Marsupials species using asexual method 28. Key: B isOphidiastergranifer (one of the species Explanation: Marsupials are any members of of starfish) and Holothuriaparvula (Sea the mammalian infraclass Marsupialia. All cucumber). Sexes are separate and fertilization extant marsupials are endemic to Australasia is external. Oviparity is common, but a and the Americas. A distinctive few echinoderms are viviparous. characteristic common to these species is that Educational Objective: To know about the most of the young are carried in a pouch. important species, and their unique features. Educational Objective: To know about the

2.15 Ameoba important species, and their unique features.

26. Key: C 3.3 Mix

Explanation: Assertion is correct and reason is 29. Key: B the correct explanation of assertion. Amoeba is a Explanation: Tape worms are hermaphrodite unicellular animal. It reproduce by the process (both male and female reproductive organ of asexual reproduction. Fission is one of the present). processes of asexual reproduction, where Round worms are unisexual (separate sex). amoeba divides in half to produce off spring Filaria is caused by Wuchereriabancrofti which Educational Objective: To know about the is a parasitic nematode. important species, and their unique features. Guinea worm is a long and very thin nematode, 3. Miscellaneous not an annelid. 3.1 Scientific Names Educational Objective: To know about the

27. Key: C important species, and their unique features.

Explanation: The onager also known as 3.4 Keystone Species hemione or Asiatic wild ass, is a species of the 30. Key: A family Equidae (horse family) native to Asia. A Explanation: A keystone species is a species member of the subgenus Asinus, the onager was that play a critical role in maintaining the described and given its binomial name by structure of an ecological community and whose German zoologist Peter Simon Pallas in 1775. impact in the community is greater than world The barasingha (Rucervusduvauceliisyn .Cervusduvaucelii), also called swamp deer, is a

www.laex.in Page No. 49 https://elearn.laex.in

n

www.laex.in

Environment UPSC

Previous Year Questions

be expected based on its relative abundance or 4. Conservation Status and total biomass. Conservation Efforts Educational Objective: to know about the 4.1 IUCN important species, and their unique features. 34. Key: B And ecological importance. Explanation: IUCN is a voluntary organization,

3.5 Insects not an agency of United Nations. Even if you

31. Key: D knew this much, you could eliminate all other Explanation: Instead of lungs, insects options to select B. Some facts about IUCN: breathe with a network of tiny tubes called Founded in 1948 as the world’s first global tracheae. Air enters the tubes through a row of environmental organisation holes along an insect's abdomen. The air then Today the largest professional global diffuses down the blind-ended tracheae. conservation network Educational Objective: to know about the A leading authority on the environment and important species, and their unique features. sustainable development More than 1,200 member organizations 3.6 Mammals including 200+ government and 900+ non- 32. Key: B government organizations Explanation: Sea cow is a mammal. It is a A neutral forum for governments, NGOs, herbivore and it grows up to 9 meters. It looks scientists, business and local communities to like a large seal. Sea horse is a fish not a find practical solutions to conservation and mammal. Sea lion is a mammal. It is a development challenges carnivore. It is also known as eared seal. Thousands of field projects and activities around Educational Objective: to know about the the world CITES was drafted as a result of a important species, and their unique features. resolution adopted in 1963 at a meeting of

3.7 Pollinating Agents members of the International Union for Conservation of Nature (IUCN). States (countries) 33. Key: D adhere voluntarily to the agreement. Statesthat Explanation: Pollinating agents are animals have agreed to be bound by the Convention such as insects, birds, and bats; water; wind; (‘joined’ CITES) are known as Parties. and even plants themselves, when self- Although CITES is legally binding on the Parties pollination occurs within a closed – in other words they have to implement the flower. Pollination often occurs within a species. Convention – it does not take the place of When pollination occurs between species it can national laws. Rather it provides a framework to produce hybrid offspring in nature and in plant be respected by each Party, which has to adopt breeding work. its own domestic legislation to ensure that Educational Objective: To know about the CITES is implemented at the national level. biodiversity.

www.laex.in Page No. 50 https://elearn.laex.in

n

www.laex.in

Environment UPSC

Previous Year Questions

Educational Objective: To know about the It is listed in Vulnerable (VU) Category – IUCN. It international organizations, their environmental is the State animal of Madhya Pradesh and Uttar initiatives and their work towards conservation Pradesh. They prefer tall grasslands and open habitats. 35. Key: B They found in swampland and a variety of forest Explanation: IUCN is an NGO. It publishes Red types ranging from dry to moist deciduous to data book which contains a list of ‘Threatened evergreen. They also found in grassy floodplains, species’ (vulnerable, endangered and critically wooded areas, and found near water bodies endangered). Their Range includes central and northern India Educational Objective: To know about the and southern Nepal international organizations, their environmental Educational Objective: To know about initiatives and their work towards conservation. Biodiversity of India, their range and IUCN status 4.2 Endangered Species

4.3 In-situ method 36. Key: A 38. Key: B Explanation: Red Panda and Asiatic Wild Ass, Explanation: are endangered species. Educational Objective: To know about the different types of species and their conservation status

37. Key: D Explanation: Gharial Critically Endangered— IUCN Red List Gharials once inhabited all the major river systems of the Indian Subcontinent, from the Educational Objective: To know about the Irrawaddy River in the east to the Indus River in methods of conservation of flora

the west. Their distribution is now limited to 4.4 M- Stripe only 2% of their former range. It now includes 39. Key: B Girwa River, Chambal River, Ken River, Son Explanation: The android-based monitoring River, Mahanadi River etc., software M-STrIPES will be used across all the Leatherback sea turtle Tiger Reserves of the country. It is listed in Vulnerable category of IUCN and also in CITES Appendix I species. Educational Objective: Initiatives, and It is the largest of all living turtles. They can be technology usage in the conservation of wildlife

found primarily in the open ocean and nesting 5. Sustainable Development areas are in the tropics. 40. Key: B Swamp Deer or Barasingha Explanation: The Partnership for Action on Green Economy (PAGE), a UN mechanism to assist countries transition towards greener and

www.laex.in Page No. 51 https://elearn.laex.in

n

www.laex.in

Environment UPSC

Previous Year Questions

more inclusive economies and to deliver Educational Objective: To know about the a more coordinated and effective response to international environmental initiatives. And member state initiatives, emerged at the United their work towards conservation. Nations Conference on Sustainable Development 6. Bio- Diversity Global Efforts 2012, Rio de Janeiro. 44. Key: B Educational Objective:To know about the Explanation: It is a NGO, not a bureau under international organizations, their environmental UNEP. TRAFFIC is governed by the TRAFFIC initiatives and their work towards conservation.

Committee, a steering group composed of 41. Key: A members of TRAFFIC’s partner organizations, Explanation: Agenda 21 is a non-binding, WWF and IUCN. It is working globally on Wildlife voluntarily implemented action plan of the trade monitoring network. United Nations with regard to sustainable It specializes in investigating and analysing development. wildlife trade trends, patterns, impacts and It is a product of the Earth Summit (UN drivers to provide the leading knowledge base on Conference on Environment and Development) trade in wild animals and plants. held in Rio de Janeiro, Brazil, in 1992. Educational Objective: to know about the Educational Objective: to know about the international environmental initiatives international organizations, their environmental 45. Key: C initiatives and their work towards conservation.

Explanation: The Economics of Ecosystems and 42. Key: B Biodiversity (TEEB) is a global initiative focused Explanation: Club of Rome talked about “limits on “making nature’s values visible”. Its principal to growth. The Sustainable Development Goals objective is to mainstream the values of were first proposed in 2012 Rio plus 20 biodiversity and ecosystem services into Educational Objective: To know about the decision-making at all levels. It aims to achieve international environmental initiatives And their this goal by following a structured approach to work towards conservation. valuation that helps decision-makers recognize

43. Key: C the wide range of benefits provided by Explanation: Earth Hour is a worldwide ecosystems and biodiversity, demonstrate their movement for the planet organized by the World values in economic terms and, where Wide Fund for Nature (WWF). The event is held appropriate, capture those values in decision- worldwide annually encouraging individuals, making communities, households and businesses to Educational Objective:To know about the turn off their non-essential lights for one hour, international environmental initiatives from 8:30 to 9:30 p.m. to raise the awareness 46. Key: C about the climate change and the need to save Explanation: Bird Life international is a global the planet. partnership of conservation organisations that strives to conserve birds, their habitats and

www.laex.in Page No. 52 https://elearn.laex.in

n

www.laex.in

Environment UPSC

Previous Year Questions

global biodiversity, working with people towards It was hunted for its skin throughout its range sustainability in the use of natural resources. It up to the 1970s, and is threatened by illegal is the World’s largest partnership of killing and habitat loss. conservation organisations, with over 120 Educational Objective: To know about the partner organizations 120 partner organizations. different types of species.

Together the BirdLife Partnership forms the 48. Key: D leading authority on the status of birds, their Breast Bone habitats & the issues and problems affecting The sternum or breastbone is a long flat bone bird life. located in the central part of the chest. It BirdLife’s Important Bird and Biodiversity Area connects to the ribs via cartilage and forms the front of the rib cage, thus helping to protect the (IBA) Programme aims to identify, monitor and heart, lungs, and major blood vessels from protect a global network of IBAs for the injury. conservation of the world’s birds and other Collarbone: wildlife. The clavicle, or collarbone, is a long bone that Educational Objective: to know about the serves as a strut between the shoulder blade international environmental organizations. and the sternum (breastbone). There are two

7. Miscellaneous clavicles, one on the left and one on the right.

47. Key: A The clavicle is the only long bone in the body Explanation: The saltwater crocodile that lies horizontally. (Crocodylusporosus) is a common species Shoulderblade: throughout the Andaman and Nicobar Islands. The scapula, or shoulder blade, is a large It can be encountered in open sea, near the triangular-shaped bone that lies in the upper shore, mangrove creeks, fresh water rivers and back in swamps. Kneecap: The great Himalayan range has a very The patella, also known as the kneecap, is a flat, interesting variety of fauna that includes the circular-triangular bone which articulates with wild sheep and goats, markhor, ibex, shrew and the femur (thigh bone) and covers and protects tapir. the anterior articular surface of the knee joint. Tapirs are mainly found in America, though Educational Objective: To know about various there are species in Southeast Asia (Malay). important bones in the Human body. The saltwater crocodile has been listed as Least Concern on the IUCN Red List since 1996.

www.laex.in Page No. 53 https://elearn.laex.in

n

www.laex.in

Environment UPSC

Previous Year Questions

www.laex.in Page No. 54 https://elearn.laex.in

n

www.laex.in

Environment UPSC

Previous Year Questions

c. 1 and 3 only V. Ecology and Environment d. 1, 2 and 3

1. Ecosystem 1.3 Services

1.1 Productivity 4. The Millennium Ecosystem Assessment

1. Which one of the following is the correct describes the following major categories sequence of ecosystems in the order of of ecosystem services- provisioning, decreasing productivity? supporting, regulating, preserving and a. Oceans, lakes, grasslands, mangroves cultural. Which one of the following is b. Mangroves, oceans, grasslands, lakes supporting service? c. Mangroves, grasslands, lakes, oceans a. Production of food and water d. Oceans, mangroves, lakes, grasslands b. Control of climate and disease c. Nutrient cycling and crop pollination 2. In the context of ecosystem d. Maintenance of diversity productivity, marine upwelling zones

are important as they increase the 1.4 Meaning/Definition

marine productivity by bringing the 5. Which one of the following is the best 1) Decomposer microorganisms to the description of the term ‘Ecosystem'? surface. a. A community of organisms interacting with 2) Nutrients to the surface. one another 3) Bottom-dwelling organisms to the surface. b. That part of the Earth which is inhabited Which of the statements given above is/are by living organisms correct? c. A community of organisms together with a. 1 and 2 the environment in which they live b. 2 only d. The flora and fauna of a geographical area c. 2 and 3 1.5 Terminology d. 3 only

6. Which one of the following terms 1.2 Terrestrial Ecosystem describes not only the physical

3. Which of the following leaf spaceoccupied by an organism, but also modifications occurs/occur in desert its functional role in the community of areas to inhibit water loss? organisms? 1) Hard and waxy leaves a. Ecotone b. Ecological niche 2) Tiny leaves or no leaves c. Habitat 3) Thorns instead of leaves d. Home range Select the correct answer using the codes given below. a. 1 and 2 only b. 2 only

www.laex.in Page No. 55 https://elearn.laex.in

n

www.laex.in

Environment UPSC

Previous Year Questions

2. Aquatic Ecosystem 9. With reference to a conservation 2.1 Wetland organization called ‘Wetlands International’, which of the following 7. With reference to the wetlands of India, statements Is/are correct? consider the following statements: 1) It is an intergovernmental organization 1) The country's total geographical area under formed by the countries which are the category of wetlands is recorded more signatories to Ramsar Convention. in Gujarat as compared to other States. 2) It works at the field level to develop and 2) In India, the total geographical area of mobilize knowledge, and use the practical coastal wetlands is larger than that of experience to advocate for better policies. inland wetlands. Select the correct answer using the codes Which of the statements given above is/are given below. correct? a. 1 only a. 1 only b. 2 only b. 2 only c. Both 1 and 2 c. Both 1 and 2 d. Neither 1 nor 2 d. Neither 1 nor 2 10. Consider the following pairs 8. Consider the following statements Wetlands Confluence of rivers 1. Under Ramsar convention, it is mandatory 1. Harike Wetlands : Confluence of Beas on the part of the Government of India to and Satluj/Sutlej protect and conserve all the wetlands in 2. Keoladeo Ghana : Confluence of the territory of India. National Park Banas and Chambal 2. The Wetlands (Conservation and 3. Kolleru Lake : Confluence of Musi Management) Rules, 2010 were framed by and Krishna the Government of India based on the Which of the above pairs is / are correctly recommendations of Ramsar convention. matched? 3. The Wetlands (Conservation and a. 1 only Management) Rules, 2010 also encompass b. 2 and 3 only the drainage area or catchment regions of c. 1 and 3 only the wetlands as determined by the d. 1, 2 and 3 authority. 11. If a wetland of international importance is Which of the statements given above is/are brought under the 'Montreux Record', correct? what does it imply? a. 1 and 2 only a. Changes in ecological character have b. 2 and 3 only occurred, are occurring or are likely to occur c. 3 only in the wetland as a result of human d. 1,2 and 3 interference

www.laex.in Page No. 56 https://elearn.laex.in

n

www.laex.in

Environment UPSC

Previous Year Questions

b. The country in which the wetland is located 3) The density of ocean water would should enact a law to prohibit any human drastically decrease. activity within five kilometers from the edge Select the correct answer using the codes of the wetland given below: c. The survival of the wetland depends on the a. 1 and 2 only cultural practices and traditions of certain b. 2 only communities living in its vicinity and c. 3 only therefore the cultural diversity therein d. 1, 2 and 3

should not be destroyed 14. There is a concern over the increase in d. It is given the status of 'World Heritage harmful algal blooms in the seawaters of Site' India. What could be the causative

12. India is a party to the Ramsar factors for this phenomenon? Convention and has declared many 1) Discharge of nutrients from the estuaries. areas as Ramsar Sites. Which of the 2) Run-off from the land during the monsoon. following statements best describes as 3) Upwelling in the seas. to how we should maintain these sites Select the correct answer from the codes in the context of this Convention? given below: a. Keep all the sites completely inaccessible to man so that they will not be exploited a. 1 only b. Conserve all the sites through ecosystem b. 1 and 2 only approach and permit tourism and c. 2 and 3 only recreation only d. 1, 2 and 3

c. Conserve all the sites through ecosystem 3. Functions of an Ecosystem approach for a period without any 3.1 Carbon Cycle

exploitation, with specific criteria and specific period for each site, and then allow 15. Which of the following adds/add carbon sustainable use of them by future dioxide to the carbon cycle on the generations planet Earth? d. Conserve all the sites through ecosystem 1) Volcanic action approach and allow their simultaneous 2) Respiration sustainable use. 3) Photosynthesis

2.2 Miscellaneous 4) Decay of organic matter Select the correct answer using the code 13. What would happen if phytoplankton of given below. an ocean is completely destroyed for a. 1 and 3 only some reason? b. 2 only 1) The ocean as a carbon sink would be c. 1, 2 and 4 only adversely affected. d. 1, 2, 3 and 4 2) The food chains in the ocean would be

adversely affected.

www.laex.in Page No. 57 https://elearn.laex.in

n

www.laex.in

Environment UPSC

Previous Year Questions

16. Human activities in the recent past 2) Fungi have caused the increased 3) Bacteria concentration of carbon dioxide in the Select the correct answer using the codes atmosphere, but a lot of it does not given below. remain in the lower atmosphere because a. 1 only of b. 2 and 3 only 1) Its escape into the outer stratosphere. c. 1 and 3 only 2) The photosynthesis by phytoplankton in d. 1, 2 and 3 the oceans 19. A pesticide which is a chlorinated 3) The trapping of air in the polar ice caps. hydrocarbon is sprayed on a food crop. Which of the statements given above is/ are The food chain is: Food crop-Rat- Snake- correct? Hawk. In this food chain, the highest a. 1 and 2 concentration of the pesticide would b. 2 only accumulate in which one of the c. 2 and 3 following? d. 3 only a. Food crop

3.2 Food Chain b. Rat

17. With reference to food chains in c. Snake ecosystems, consider the following d. Hawk

statements: 3.3 Ecological Succession 1) A food chain illustrates the order in which 20. In the grasslands, trees do not replace a chain of organisms feed upon each other the grasses as a part of an ecological 2) Food chains are found within the succession because of populations of aspecies a. Insects and fungi 3) A food chain illustrates the number of each b. Limited sunlight and paucity of nutrients organism which are eaten by others c. Water limits and fire Which of the statements given above is/are d. None of the above correct? 21. Lichens, which are capable of initiating a. l only ecological succession even on a bare b. 1 and 2 only rock, are actually a symbiotic c. 1,2 and 3 association of d. None

a. Algae and bacteria 18. With reference to the food chains in b. Algae and fungi ecosystems, which of the following c. Bacteria and fungi kinds of organism is/are known as d. Fungi and mosses decomposer organism/organisms? 1) Virus

www.laex.in Page No. 58 https://elearn.laex.in

n

www.laex.in

Environment UPSC

Previous Year Questions

oxygen creating abnoxic conditions and Key and Explanations consequently dead zones. Bringing

1. Ecosystem organisms from depth increases 1.1 Productivity competition resulting in declining productivity. 1. Key: C Educational Objective: To understand why only Explanation: Productivity is the production of some locations are the largest fishing grounds in new biomass by an ecosystem. the world and reasons for their distribution Different ecosystem have different rate of productivity which depend on various factors 1.2 Terrestrial Ecosystem like water, nutrients, temperature etc. 3. Key: D In the given ecosystems mangrove have highest Explanation: The plants in desert survive the productivity due to it having high species extreme climates by xerophytic adaptations richness as it represents ecotone between which includes various modification in roots, terrestrial and marine ecosystem. Then comes stem and leaves. Modification of leaves is the the grasslands which represents ecotone frequent adaptation between tropics and desert but less productive  Hard and waxy leaves prevent excess than mangroves due to low moisture content. transpiration and also protects leaves from Lake comes third as its productivity various on direct heat of sun level of nutrients with eutrophic being more  Tiny leaves and thorny leaves also regulate productive. Oceans are least among the four excess transpiration and also inturn inspite of being large in area because only up to protects plants from cattle. 250 m of ocean is productive where sunlight is  Hence all are helpful for survival of plants available called photic zone. in desert climate Hence option c is the right sequence Educational Objective: To understand how Educational Objective: To study salient fauna and flora survive in extreme desert features of different ecosystems and evaluating conditions. their contribution to world biomass. 1.3 Services

2. Key: B 4. Key: C Explanation: Marine upwelling zones are one of Explanation: Supporting services are the the most productive fishing grounds in the world services which are responsible for production of (Japanese coast, Newfoundland) because in this all other ecosystem services, hence option c is zone the hot waters are replaced by the lower correct as nutrient cycling and pollination helps cold waters due to tidal action which bring the in production of food which is a provisioning vast nutrients from ocean deep to the surface service of ecosystem. enhancing species richness and productivity.  Control of climate and disease is a  Statements 1 and 3 are incorrect because regulating service as it regulates the microorganisms at surface decreases ecosystem processes.

www.laex.in Page No. 59 https://elearn.laex.in

n

www.laex.in

Environment UPSC

Previous Year Questions

 Maintenance of diversity is a cultural Habitat is the place or environment where a service as it increases species richness and plant or animal naturally or normally lives and grows. in turn creates recreational value. Educational Objective: To know about Educational Objective: To understand the important ecological terms such as Ecological importance of ecosystem in the day today life of niche, Ecotone, Habitat etc., human beings. 2. Aquatic Ecosystem 2.1 Wetland 7. Key: C Explanation: Wetlands are areas intermediate in character between deep water and terrestrial habitats, also transitional in nature, and often located between them.  Gujarat has largest wetland area followed by Andhra Pradesh, Uttar Pradesh, west

Bengal and in top five. 1.4 Meaning/Definition  Coastal wetlands occupy more than 50% of 5. Key: C wetlands in India prominent among them Explanation: Option a is incorrect as it is are chilika lake ( Orissa), Pulicatetc missing the abiotic component i.e environment Educational Objective: To study wetland  Part of earth on which we live is biosphere distribution in India and it’s ecological hence option b also incorrect. importance.  Option c is the best and most complete 8. Key: C definition of ecosystem (interaction Explanation: The Convention on Wetlands between the biotic (living organisms) and [waterfowl convention] is an intergovernmental abiotic (Soil, climate etc)). treaty that provides the framework for national Educational Objective: To understand different action and international cooperation for the ecological terms like ecology, ecosystem, ecotone conservation and wise use of wetlands and their etc., and differentiate them

resources. 1.5 Terminology  Statement 1 is not correct as it is 6. Key: B mandatory to protect at least wetlands of Ecological Niche refers to the unique functional international importance and is role and position of a species in its habitat or voluntary on part of government with ecosystem. A niche can also be described as the interactions, a species has with its biotic and respect to other wetlands. abiotic environment. Hence, option B is correct.  Statement 2 is incorrect as rules are made Ecotone is a zone of junction or a transition area independent of Ramsarconvention for between two biomes (diverse ecosystems). regulating various activities with respect to all wetlands in the country.

www.laex.in Page No. 60 https://elearn.laex.in

n

www.laex.in

Environment UPSC

Previous Year Questions

 Hence statement 3 is only correct answer  Kollerulake is one of the largest fresh water as wetland rules 2010 have been made lake in India and largest shallow water lake stringent by encompassing the drainage between deltas of Krishna and Godavari. and catchment areas of wetlands. Hence only the first option is correctly Educational Objective: To study wetlands and matched. related rules and convention like Ramsar Educational Objective: To know the geography convention which is only convention for wetland and features of wetlands of International protection Importance in India.

9. Key: B 11. Key: A Explanation: Statement 1 in incorrect because Explanation: Montreux Record is the principal wetlands International is a not for profit tool under the Ramsar Convention, is a register organization working in the field of wetland of wetland sites on the List of Wetlands of conservation. International Importance It highlights those  Wetlands International’s work ranges from sites where adverse changes in ecological research to community based field projects character have occurred, are occurring, or are to advocacy and engagement with likely to occur as a result of technological governments corporates and various developments, pollution or other human conventions. interference and which are therefore in need of Educational Objective: To study about various priority conservation attention. It is maintained organization associated with wetland protection as part of the Ramsar List. and management. Montreux Record are employed to identify priority sites for positive national and 10. Key: A international conservation attention. Sites may Explanation: Harike wetland also called “Hari- be added to and removed from the Record only ke-Pattan” is the largest wetland in north India with the approval of the Contracting Parties in constructed in 1953 by erecting headworks at which they lie. the confluence of Beas and Sutlej just south of Wetlands from India in Montreux record: Harike village. Loktaklake and Keoladeo national park  Keoladeo Ghana National Park is a man Educational Objective: To know about the made wetland and one of national parks in wetlands in montreux record from India and India. Initially it was a depression which steps taken to address the issues with them

was flooded to form wetland when 12. Key: D Maharaja constructed Arjun Bund at the

confluence of rivers Gambhir and Explanation: The Ramsar Convention’s aim is Banganga in 1750s. In 1985 the park was to develop and maintain an international declared world heritage convention. network of wetlands which are important for the conservation of global biological diversity and for sustaining human life through the

www.laex.in Page No. 61 https://elearn.laex.in

n

www.laex.in

Environment UPSC

Previous Year Questions

maintenance of their ecosystem components, 14. Key: A processes and benefits. Explanation: Algal blooms are the excessive

The Ramsar Convention is an international growth of microscopic algae due to addition of treaty for the conservation and wise use of nutrients which as a catalyst for multiplying wetlands. algae. Most algal blooms are not harmful but  It is named after the Iranian city of Ramsar, some produce toxins and do affect fish, birds, on the Caspian Sea, where the treaty was marine mammals and humans. The toxins may signed on 2 February 1971. also make the surrounding air difficult to  Known officially as ‘the Convention on breathe. These are known as Harmful Algal Blooms (HABs). Wetlands of International Importance  Runoff from the land during monsoon may especially as Waterfowl Habitat’ (or, more empty muddy water into sea but it may not recently, just ‘the Convention on Wetlands’), lead to algal growth as it may not contain it came into force in 1975. the nutrients required for there growth.

2.2 Miscellaneous Hence statement 2 is incorrect.

13. Key: A  Upwelling of seas bring the nutrients to the Explanation: Phyto-planktons are the primary surface increasing productivity of marine producers of the ocean ecosystem and form the organisms. Hence statement 3 is also base of food chain. incorrect.  Phyto-planktons being the primary Education Objectives: Studying the concept of producers absorb great quantity of carbon- eutrophication and it effect on algal bloom. di-oxide from the atmosphere for 3. Functions of an Ecosystem photosynthesis, thus acting as carbon 3.1Carbon Cycle sink.

 Thus complete destruction of 15. Key: C Phytoplanktons affect the ocean as carbon Explanation: Volcanic action is the eruption of sink and also food chain will be affected interior of the earth in the form of magma during destroying the entire ocean ecosystem. such various gases like SO2, Carbon-di-oxide Hence statement 1 and 2 are correct. are released into atmosphere.  Statement 3 is wrong as density of ocean  Respiration is the process of breathing by water is dependent on the amount of living organisms which release carbon-di- dissolved salts and rate of evaporation. oxide and intake oxygen for their metabolic Education Objectives: studying the basic activities. building blocks of ocean ecosystem the phyto  Photosynthesis is the ultimate producer of planktons. food for all living organisms by the primary producers which produce glucose by

consuming carbon-di-oxide and releasing

oxygen in the process.

www.laex.in Page No. 62 https://elearn.laex.in

n

www.laex.in

Environment UPSC

Previous Year Questions

 Decaying of organic matter release co2  Food chain indicates only the source and which is stored as biomass during the life direction of flow of energy and not the of the organism. number of organisms eaten hence  Thus option 1, 2 and 4 are process statement 3 is incorrect. responsible for releasing co2 back into Educational Objective: To understand the flow carbon cycle. of energy in an ecosystem through the concept Education Objectives: Understanding the of food chain and food web.

concept of carbon cycle 18. Key: B

16. Key: B Explanation: Viruses are biological agents that Explanation: Statement 1 is incorrect because reproduces inside the living cells only and hence carbon-di-oxide is found only upto 90 km from they cannot exist without host and hence are earth surface and does not extend to outer not decomposers. stratosphere.  Fungi are the primary decomposers in the  Phyto planktons are the primary producers ecosystem which grow on dead or rotten of marine ecosystem and they absorb great organic matter as Saprophytes or as amount of co2 for their photosynthesis parasites in host cell. from the lower atmosphere hence co2 in Ex: molds, mushroom lower atmosphere is readily absorbed by  Bacteria are non-chlorophyllous micro- them decreasing their quantity and acting organisms which lead saprophytic or as lungs. Hence statement 2 is correct parasitic existence mainly. Saprophytic are  Trapping of air in polar ice caps contain beneficial and used in many industries. large amount of co2 of historic emissions Educational Objective: To study about and not recent ones hence statement 3 is saprophytes and role theypaly in cleaning the incorrect. surroundings Educational Objective: To study about various 19. Key: D methods of Carbon sequestration and role Explanation: Bio-magnification is the process of played by Phyto planktons in it. accumulation of pollutants as one moves from

3.2 Food Chain lower trophic to higher trophic levels.

17. Key: A Bio-accumulation is the process of increase in Explanation: Food chain represents the concentration of pollutant from environment into sequence of eaten and being eaten hence first organism in food chain. statement 1 is correct. In the given list of organisms the food chain  Food chain represents the entire sequence sequence is Food-rat-snake-hawk of organisms from primary producers to Hence hawk being the tertiary consumers has tertiary consumers of different species. the highest concentration of pollutant. Ex: Plant-caterpillar-lizard-snake Hence statement 2 is incorrect.

www.laex.in Page No. 63 https://elearn.laex.in

n

www.laex.in

Environment UPSC

Previous Year Questions

Educational Objective: To understand bio- VI Global Environmental Issues accumulation and bio-magnification and difference between them. 1. Green House Gases 1.1 Carbon Dioxide 3.3 Ecological Succession

1. The increasing amount of carbon 20. Key: C dioxide in the air is slowly raising the Explanation: Grasslands represent the ecotone temperature of the atmosphere, because between the tropical forests and deserts. In it absorbs: grasslands tress do not replace grasses because a. The water vapour of the air and retains its of low moisture and high temperatures which heat results in frequent fires in grasslands. b. The ultraviolet part of the solar radiation Insects and fungi also cannot survive in c. All the solar radiations extremes of grasslands. d. The infrared part of the solar radiation

Hence option 3 is the correct answer. Educational Objective: To study the fauna and 1.2 Methane flora of grasslands and features influencing it. 2. Consider the following: 1) Rice fields 21. Key: B 2) Coal mining Explanation: Lichens represents a group of 3) Domestic Animals greyish green plants which grow on rocks, tree 4) Wetlands trunks, dead wood etc. It represents a symbiotic Which of the above are sources of methane, a relation between algae and fungi. major greenhouse gas?  In this unique relationship algae a. 1 and 4 only manufactures food for fungi and fungi in b. 2 and 3 only turn absorbs and retains water which c. 1, 2 and 3 only keeps algal cells moist. d. 1, 2, 3 and 4

 They are most common in wetlands, rare in 2. Impact of Climate Change streams and rivers and absent in ground 3. The scientific view is that the increase water. in global temperature should not exceed Educational Objective: To understand 2°C above pre-industrial level. If the interaction between various organisms for global temperature increases beyond 3°C survival and functioning of ecosystem above the pre-industrial level, what can

be its possible impact/impacts on the

world?

1) Terrestrial biosphere tends toward a net

carbon source.

2) Widespread coral mortality will occur.

www.laex.in Page No. 64 https://elearn.laex.in

n

www.laex.in

Environment UPSC

Previous Year Questions

3) All the global wetlands will permanently b. 3 only disappear. c. 1 and 3 only 4) Cultivation of cereals will not be possible d. 1,2 and 3 anywhere in the world. 4. Ozone Depletion Select the correct answer using the code 6. Which one of the following is associated given below with the issue of control and phasing a. l only out of the use of ozone-depleting b. 1 and 2 only substances? c. 2, 3 and 4 only a. Bretton Woods Conference d. 1,2,3 and 4

b. Montreal Protocol 3. Mitigation c. Kyoto Protocol 3.1 Cloud thinning + Aerosol Injection d. Nagoya Protocol

4. In the context of which of the following 7. Consider the following statements: do some scientists suggest the use of Chlorofluorocarbons, known as ozone- cirrus cloud thinning technique and the depleting substances, are used. injection of Sulphate aerosol into 1) In the production of plastic foams stratosphere? 2) In the production of tubeless tyres a. Creating the artificial rains in some regions 3) In cleaning certain electronic components b. Reducing the frequency and intensity of 4) As pressurizing agents in aerosol cans tropical cyclones Which of the statements given above is/are c. Reducing the adverse effects of solar wind correct? on the earth a. 1, 2 and 3 only d. Reducing the global warming b. 4 only

3.2 Carbon Sequestration Sites c. 1, 3 and 4 only d. 1, 2, 3 and 4

5. In the context of mitigating the impending global warming due to 8. The formation of ozone hole in the anthropogenic emissions of carbon Antarctic region has been a cause, of dioxide, which of the following can be concern. What could be the reason for the potential sites for carbon the formation of this hole? sequestration? a. Presence of prominent tropospheric 1) Abandoned and uneconomic coal seams turbulence; and inflow of 2) Depleted oil and gas reservoirs chlorofluorocarbons 3) Subterranean deep saline formations b. Presence of prominent polar front and Select the correct answer using the code stratospheric clouds; and inflow of given below chlorofluorocarbons a. 1 and 2 only

www.laex.in Page No. 65 https://elearn.laex.in

n

www.laex.in

Environment UPSC

Previous Year Questions

c. Absence of polar front and stratospheric absorb infrared photons. CO2 molecules can clouds; and inflow of methane and vibrate in ways that simpler nitrogen and oxygen chlorofluorocarbons. molecules cannot, which allows CO2 molecules d. Increased temperature at polar region due to capture the IR photons. to global warming Water vapour in the atmosphere does act as a greenhouse gas, however this is not because of 5. Ocean Acidification its absorption by carbon dioxide. Air in general 9. The acidification of oceans is has water vapour which traps heat and provides increasing. Why is this phenomenon a a warming effect. It is the infrared part of solar cause of concern? radiation which carries heat. Even if Carbon 1) The growth and survival of calcareous Dioxide trapped phytoplankton will be adversely affected. Ultra Violet light it would not lead to rise in 2) The growth and survival of coral reefs will temperature. Therefore (b) is a made up be adversely affected. statement. 3) The survival of some animals that have If Carbon Dioxide were to trap all the Solar phytoplanktonic larvae will be adversely Radiation, the Earth would have had a cooling affected. effect instead of rise in temperature. Hence

4) The cloud seeding and formation of clouds option C is incorrect as well. will beadversely affected. Educational Objective: Test candidate's Which of statements given above is / are knowledge of harmful effects of carbon dioxide correct? on the environment. a. 1,2 and 3 only 1.2 Methane b. 2 only 2. Key: D c. 1 and 3 only Explanation: Wetlands are the largest source d. 1,2,3 and 4 for Methane emission, as bacteria decompose

organic materials in the absence of oxygen. Key and Explanations Domestic Livestock such as cattle, buffalo, sheep, goats and camels produce large amounts 1.1 Carbon Dioxide of methane as part of their normal digestive 1. Key: D process. Explanation: Molecules of carbon dioxide (CO ) 2 Fermentation process in Rice Fields produces can absorb energy from infrared (IR) radiation. methane gas too. Methane is released as a direct This ability to absorb and re-emit infrared result of the physical process of Coal energy is what makes CO an effective heat- 2 extraction. trapping greenhouse gas. Not all gas molecules Educational Objective: Learn about sources of are able to absorb IR radiation. For example, major Greenhouse gases. Nitrogen (N2) and Oxygen (O2), which make up more than 90% of Earth's atmosphere, do not

www.laex.in Page No. 66 https://elearn.laex.in

n

www.laex.in

Environment UPSC

Previous Year Questions

2. Impact of Climate Change temperatures which is humanly impossible and 3. Key: B hence is incorrect. Explanation: Impact of rise in temperature Artificial rains are created by spraying Dry above 2 degree from pre industrial levels: Ice(solid carbon dioxide), Silver Iodide, Salt powder etc. on the rain bearing clouds. Hence it is incorrect. Hence statement 1 and 2 are correct. Educational Objective: Test candidate's Statement 3 states that all the global wetlands knowledge of various methods being suggeted to will permanently disappear, which is an extreme reduce the effect of global warming. statement and can be eliminated easily. 3.2 Carbon Sequestration Sites Similar to statement 3, statement 4 is also 5. Key: D extreme which says cultivation of cereals will not Explanation: Carbon sequestration or carbon be possible and can be eliminated. dioxide removal is the long-term removal, Educational Objective: Understand the impact capture or sequestration of carbon dioxide from of Global warming and climate change.

the atmosphere to slow or reverse atmospheric 3.1 Cloud thinning + Aerosol Injection CO2 pollution and to mitigate or reverse global 4. Key: D warming. The carbon that is captured can be Explanation: Cirrus cloud thinning is a stored in geological formations such as deep proposed form of climate engineering. Cirrus saline aquifers, basalt formations, unmineable clouds are high cold ice that, like other clouds, coal seams and depleted oil or gas reservoirs. both reflect sunlight and absorb warming Hence all the given options are correct. infrared radiation. However, they differ from Educational Objective: To learn various other types of clouds in that, on average, methods that can be employed to reduce the infrared absorption outweighs sunlight release of various Greenhouse Gases. reflection, resulting in a net warming effect on 4. Ozone Depletion the climate. Therefore, thinning or removing 6. Key: B these clouds would reduce their heat trapping Explanation: Bretton Woods Conference: This capacity, resulting in a cooling effect on Earth's conference led to the establishment of the World climate. This could be a potential tool to reduce Bank and IMF. Hence option (a) is incorrect. anthropogenic global warming. Cirrus cloud Montreal Protocol: The Montreal Protocol is an thinning is an alternative category of climate international treaty designed to protect the engineering, in addition to solar radiation ozone layer by phasing out the production of management and greenhouse gas removal. numerous substances that are responsible for Solar Winds have hardly any significant impact ozone depletion on earth hence it is incorrect. Kyoto Protocol: The Kyoto Protocol is an Reducing the frequency and intensity of international treaty which extends the 1992 tropical cyclones requires altering ocean water United Nations Framework Convention on

www.laex.in Page No. 67 https://elearn.laex.in

n

www.laex.in

Environment UPSC

Previous Year Questions

Climate Change (UNFCCC) that commits state formed over Antarctica. These polar parties to reduce greenhouse gas emissions. stratospheric clouds provide surface on which Nagoya Protocol: The Nagoya Protocol is about chlorine nitrate formed (reaction iv) gets Access to Genetic Resources and the Fair and hydrolysed to form hypochlorous acid (reaction Equitable Sharing of Benefits Arising from their (vi)). It also reacts with hydrogen chloride Utilization to the Convention on Biological produced as per reaction (v) to give molecular Diversity. chlorine. Educational Objective: To learn and When sunlight returns to Antarctica in the differentiate various international treaties signed spring, the sun’s warmth breaks up the clouds in the context of saving mother earth. and HOCl and Cl2 are photolysed by sunlight, as given in reactions (viii) and (ix). The chlorine 7. Key: C radicals thus formed, initiate the chain reaction Explanation: CFCs are widely used in four for ozone depletion as described earlier. areas: aerosol propellants, solvents in the Educational Objective: Test candidate's cleaning of electronic components and metals, understanding of Ozone hole formation. refrigerants, and the blowing agents in foam plastic manufacturing. In aerosols, CFCs such 5. Ocean Acidification as trichlorofluoro methane (CCl3F, CFC-11) and 9. Key: D dichlorodifluoromethane (CCl2F2) were normally Explanation: Ocean acidification is the ongoing packed with materials like paint, insecticide, or decrease in the pH of the Earth's oceans, caused cosmetic preparations in pressurized canisters. by the uptake of carbon dioxide from the Tubeless tyre has no component which is a atmosphere. Seawater is slightly basic, and Greenhouse Gas. Hence it is the required ocean acidification involves a shift towards pH- answer. neutral conditions rather than a transition to Educational Objective: To learn different acidic conditions. This phenomenon will have an sources of Greenhouse Gases. effect on: growth and survival of calcareous phytoplankton, coral reefs and phytoplanktonic 8. Key: B larvae. Explanation: In 1980s atmospheric scientists Ocean acidification may lead to decreased cloud working in Antarctica reported about depletion formation and worsen global warming over time. of ozone layer commonly known as ozone hole Hence all the given statements are correct. over the South Pole. It was found that a unique set of conditions was responsible for the ozone hole. In summer season, nitrogen dioxide and methane react with chlorine monoxide (reaction iv) and chlorine atoms (reaction v) forming chlorine sinks, preventing much ozone depletion, whereas in winter, special type of clouds called polar stratospheric clouds are

www.laex.in Page No. 68 https://elearn.laex.in

n

www.laex.in

Environment UPSC

Previous Year Questions

VII Global Level Efforts to Fight d. 1, 2 and 3

Climate Change 3. The term ‘Intended Nationally Determined

1. UNFCCC Contribution’ is sometimes seen in the 1.1 Paris Agreement 2015 news in the context of

1. Consider the following statements: a. Pledges made by the European countries to 1) The International Solar Alliance was rehabilitate refugees from the war-affected launched at the United Nations Climate Middle East Change Conference in 2015. b. Plan of action outlined by the countries of 2) The Alliance includes all the member the world to combat climate change countries of the United Nations. c. Capital contributed by the member Which of the statements given above is / are countries in the establishment of Asian correct? Infrastructure Investment Bank a. 1 only d. Plan of action outlined by the countries of b. 2 only the world regarding Sustainable c. Both 1 and 2 Development Goals. d. Neither 1 nor 2

1.2 Kyoto Protocol

2. With reference to the Agreement at the UNFCCC Meeting in Paris in 2015, which 4. Consider the following statements: of the following statements is/are 1) Clean Development Mechanism (CDM) in correct? respect of carbon credits is one of the 1) The Agreement was signed by all the member Kyoto Protocol Mechanisms. countries of the UN and it will go into effect 2) Under the CDM, the projects handled in 2017 pertain only to the Annex-I countries. 2) The Agreement aims to limit the greenhouse Which of the statements given above is/are gas emissions so that the rise in average correct? global temperature by the end of this century a. 1 only does not exceed 2°C or even 1.5°C above pre- b. 2 only industrial levels c. Both 1 and 2 3) Developed countries acknowledged their d. Neither 1 nor 2

historical responsibility in global warming and committed to donate $ 1000 billion a 5. Regarding "carbon credits", which one of year from 2020 to help developing countries the following statements is not correct? to cope with climate change. a. The carbon credit system was ratified in Select the correct answer using the code conjunction with the Kyoto Protocol given below. b. Carbon credits are awarded to countries or a. 1 and 3 only groups that have reduced greenhouse b. 2 only gases below their emission quota c. 2 and 3 only

www.laex.in Page No. 69 https://elearn.laex.in

n

www.laex.in

Environment UPSC

Previous Year Questions

c. The goal of the carbon credit system is to c. World Summit on Sustainable limit the increase of carbon dioxide Development, Johannesburg, 2002 emission d. UN Climate Change Conference, d. Carbon credits are traded at a price fixed Copenhagen, 2009. from time to time by the United Nations 9. Which of the following statements best Environment Programme. describes "carbon fertilization"?

6. Consider the following pairs: a. Increased plant growth due to increased Termssometimes Seen in the news Their concentration of carbon dioxide in the Origin atmosphere 1) Annex-1 Countries : Cartagena Protocol b. Increased temperature of Earth due to increased concentration of carbon dioxide 2) Certified Emissions : Nagoya Protocol in the atmosphere Reductions c. Increased acidity of oceans as a result of 3) Clean Development : Kyoto Protocol increased concentration of carbon dioxide Mechanism in the atmosphere Which of the pairs given above is/ are d. Adaptation of all living beings on Earth to correctly matched? the climate change brought about by the a. 1 and 2 only increased concentration of carbon dioxide b. 2 and 3 only in the atmosphere

c. 3 only 2. Alliances or Coalitions d. 1, 2 and 3 10. With reference to ‘Global Climate 1.3 Miscellaneous Change Alliance’, which of the following 7. "Momentum for Change: Climate Neutral statements is/are correct? Now" is an initiative launched by 1. It is an initiative of the European Union. a. The Intergovernmental Panel on Climate 2. It provides technical and financial support Change to targeted developing countries to b. The UNEP Secretariat integrate climate change into their c. The UNFCCC Secretariat development policies and budgets. d. The World Meteorological Organization 3. It is coordinated by World Resources

8. The United Nations Framework Institute (WRI) and World Business Convention on Climate Change Council for Sustainable Development (UNFCCC) is an international treaty (WBCSD). drawn at Select the correct answer using the code a. United Nations Conference on the Human given below: Environment, Stockholm, 1972 a. 1 and 2 only b. UN Conference on Environment and b. 3 only Development, Rio de Janeiro, 1992 c. 2 and 3 only d. 1, 2 and 3

www.laex.in Page No. 70 https://elearn.laex.in

n

www.laex.in

Environment UPSC

Previous Year Questions

11. Consider the following statements: understand, quantify and manage 1) Climate and Clean Air Coalition (CCAC) to greenhouse gas emissions Reduce Short Lived Climate Pollutants is a b. It is an initiative of the United Nations to unique initiative of G20 group of countries. offer financial incentives to developing 2) The CCAC focuses on methane, black countries to reduce greenhouse gas emissions and to adopt eco-friendly carbon and hydrofluorocarbons. technologies Which of the statements given above is/are c. It is an inter-governmental agreement correct? ratified by all the member countries of the a. 1 only United Nations to reduce greenhouse gas b. 2 only emissions to specified levels by the year c. Both 1 and 2 2022 d. Neither 1 nor 2 d. It is one of the multilateral REDD+

12. With reference to the ‘Global Alliance initiatives hosted by the World Bank.

for Climate-Smart Agriculture (GACSA)’, 4. Funding related which of the following statements is/are 4.1 Green Climate Fund correct? 14. Which of the following statements 1) GACSA is an outcome of the Climate regarding 'Green Climate Fund' is/are Summit held in Paris in 2015. correct? 2) Membership of GACSA does not create any 1) It is intended to assist the developing binding obligations. countries in adaptation and mitigation 3) India was instrumental in the creation of practices to counter climate change. GACSA. 2) It is founded under the aegis of UNEP, Select the correct answer using the code OECD, Asian Development Bank and World Bank. given below: Select the correct answer using the code a. 1 and 3 only given below. b. 2 only a. 1 only c. 2 and 3 only b. 2 only d. 1, 2 and 3 c. Both 1 and 2

3. Miscellaneous d. Neither 1 nor 2

3.1 Green House Gas Protocol 4.2 Global Environment Facility

13. What is 'Greenhouse Gas Protocol’? 15. With reference to 'Global Environment a. It is an international accounting tool for Facility', which of the following government and business leaders to statements is/are correct?

www.laex.in Page No. 71 https://elearn.laex.in

n

www.laex.in

Environment UPSC

Previous Year Questions

a. It serves as financial mechanism for 'Convention on Biological Diversity' and 5.2 Forest Carbon Partnership Facility

‘United Nations Framework Convention on 18. With reference to 'Forest Carbon Climate Change’. Partnership Facility', which of the b. It undertakes scientific research on following statements is/are correct? environmental issues at global level 1) It is a global partnership of governments, c. It is an agency under OECD to facilitate businesses, civil society and indigenous the transfer of technology and funds to peoples. underdeveloped countries with specific aim 2) It provides financial aid to universities, to protect their environment d. Both a and b individual scientists and institutions involved in scientific forestry research to 4.3 Bio Carbon Fund Initiative develop eco-friendly and climate adaptation 16. BioCarbon Fund Initiative for technologies for sustainable forest Sustainable Forest Landscapes' is management. managed by the 3) It assists the countries in their 'REDD+ a. Asian Development Bank b. International Monetary Fund (Reducing Emissions from Deforestation c. United Nations Environment Programme and Forest Degradation+)' efforts by d. World Bank providing them with financial and technical

5. Emission Reduction assistance. 5.1 REDD+ Select the correct answer using the code given below. 17. Which of the following statements is/are a. 1 only correct? Proper design and effective implementation b. 2 and 3 only of UN REDD+Programme can significantly c. 1 and 3 only contribute to d. 1, 2 and 3

1) Protection of biodiversity 5.3 Green India Mission 2) Resilience of forest ecosystems 19. Which of the following best 3) Poverty reduction describes/describe the aim of 'Green Select the correct answer using the code India Mission' of the Government of given below. India? a. 1 and 2 only 1) Incorporating environmental benefits and b. 3 only costs into the Union and State Budgets c. 2 and 3 only thereby implementing the 'green d. 1, 2 and 3 accounting'.

www.laex.in Page No. 72 https://elearn.laex.in

n

www.laex.in

Environment UPSC

Previous Year Questions

2) Launching the second green revolution to enhance agricultural output so as to Key and Explanations

ensure food security to one and all in the 1. UNFCCC future. 1.1 Paris Agreement 2015

3) Restoring and enhancing forest cover and 1. Key: A responding to climate change by a Explanation: The International Solar Alliance combination of adaptation and mitigation (ISA) is an alliance of 121 countries initiated by measures India(Hence Statement 2, which states that all Select the correct answer using the code the members of UN are its members is given below. incorrect), most of them being sunshine a. 1 only countries, which lie either completely or partly b. 2 and 3 only between the Tropic of Cancer and the Tropic of c. 3 only Capricorn. The primary objective of the alliance d. 1, 2 and 3 is to work for efficient exploitation of solar 5.4 Desertification energy to reduce dependence on fossil fuels. This 20. What is/ are the importance/ initiative was first proposed by Indian Prime importances of the 'United Nations Minister NarendraModi in a speech in November Convention to Combat Desertification'? 2015 at Wembley Stadium, in which he referred 1. It aims to promote effective action through to sunshine countries as Suryaputra (‘Sons of innovative national programmes and the Sun’). The alliance is a treaty-based supportive international partnerships. intergovernmental organization. Countries that 2. It has a special/particular focus on South do not fall within the Tropics can join the Asia and North Africa regions, and its Secretariat facilitates the allocation of alliance and enjoy all benefits as other members, major portion of financial resources to with the exception of voting rights. these regions. Educational Objective: Learn about initiatives 3. It is committed to bottom-up approach, India has taken in moving towards greener ways encouraging the participation of local of generating electricity. people in combating the desertification. 2. Key: B Select the correct answer using the code Explanation: Though the agreement was given below. reached on consensus, it is subject to a. 1 only ratification by all the member countries. Hence b. 2 and 3 only statement 1 is incorrect. The Paris Agreement's c. 1 and 3 only d. 1, 2 and 3 long-term temperature goal is to keep the increase in global average temperature to well below 2 °C above pre-industrial levels; and to pursue efforts to limit the increase to 1.5 °C, recognizing that this would substantially reduce

www.laex.in Page No. 73 https://elearn.laex.in

n

www.laex.in

Environment UPSC

Previous Year Questions

the risks and impacts of climate change. Hence CERs earned from such projects can be counted statement 2 is correct.The Paris Agreement towards meeting the prescribed Kyoto targets. requires developed countries to raise finances Statement 1 is correct. with $100 billion per year as the floor by 2020, There is no hard and fast rule that only Annex I to help developing nations in both mitigation countries can resort to CDM, even non Annex I and adaptation activities, while other nations are countries can go for CDM as well. encouraged to provide funding voluntarily. Educational Objective: Test the candidates Hence statement 3 is incorrect. awareness of basic understanding of Clean Educational Objective: Learn about important Development Mechanism.

climate change related agreements. 5. Key: D

3. Key: B Explanation: Carbon credits and carbon Explanation: Countries across the globe markets are a component of national and adopted an historic international climate international attempts to mitigate the growth in agreement at the U.N. Framework Convention on concentrations of greenhouse gases (GHGs). One Climate Change (UNFCCC) Conference of the carbon credit is equal to one tonne of carbon Parties (COP21) in Paris in December 2015. In dioxide, or in some markets, carbon dioxide anticipation of this moment, countries publicly equivalent gases. Carbon trading is an outlined what post-2020 climate actions they application of an emissions trading approach. intended to take under the new international Greenhouse gas emissions are capped and then agreement, known as their Intended Nationally markets are used to allocate the emissions Determined Contributions (INDCs). Hence option among the group of regulated sources. The goal (b) is the correct answer. is to allow market mechanisms to drive Educational Objective: Terms in news. industrial and commercial processes in the direction of low emissions or less carbon 1.2 Kyoto Protocol intensive approaches than those used when 4. Key: A there is no cost to emitting carbon dioxide and Explanation: CDM allows an other GHGs into the atmosphere. Hence market industrialized/developed country with an mechanisms determine the cost of each carbon emission-reduction or emission-limitation credit and it is not fixed by UNEP. Therefore commitment under the Kyoto Protocol (called as option (d) is the incorrect one. Annex I Party or Annex B Party of the original Educational Objective: Understand various Kyoto Protocol signed in 1997) to implement an terminologies that are in news. emission-reduction project in any of those developing countries (which may otherwise be 6. Key: C not financially capable of undertaking such Explanation: All the terminologies: Annex I, projects), thereby earning them tradable Certified Emissions Reductions Clean Certified Emission Reduction (CER) credits, each Development Mechanism are all associated with equivalent to one tonne of CO2. The saleable

www.laex.in Page No. 74 https://elearn.laex.in

n

www.laex.in

Environment UPSC

Previous Year Questions

Kyoto Protocol itself. Hence option (c) is the Carbon Dioxide Fertilisation. It increases the correct answer. crop yields in some regions. Educational Objective: Students' familiarity It is used to improve production levels, both from a qualitative and from a quantitative point with terms associated with International of view. It is particularly suitable for cold protocols and treaties. climates and can be used for growing practically Educational Objective: Various terminologies all types of vegetables (asparagus, celery, associated with various international protocols. lettuce, tomatoes, aubergines etc.), greenhouse

fruit (strawberries) and ornamental plants. 1.3 Miscellaneous Hence, statement 1 is correct. 7. Key: C Educational Objective: To learn the concept of Explanation: The United Nations Climate Carbon fertilisation

Change secretariat has launched a new initiative 2. Alliances or Coalitions that will showcase efforts by individuals, 10. Key: A companies and governments that are achieving Explanation: The Global Climate Change real results in transitioning to climate neutrality. Alliance (GCCA) is a European Union flagship Momentum for Change: Climate Neutral. Now initiative which is helping the world's most brings together two of the secretariat’s flagship vulnerable countries to address climate change. activities that recognize leadership in tackling Having started with just four pilot projects in climate change by non-Party stakeholders. 2008, it has become a major climate initiative Educational Objective: Infosys won the that has funded over 80 projects of national, ‘Momentum for Change: Climate Neutral Now’ regional and worldwide scope in Africa, Asia, the award and hence asked by UPSC. Caribbean and the Pacific. Neither World

8. Key: B Resources Institute (WRI) nor World Business Explanation: The United Nations Framework Council for Sustainable Development (WBCSD) Convention on Climate Change is an has anything to do with GCCA. Hence statement international environmental treaty adopted on 9 3 is incorrect. May 1992 and opened for signature at the Earth Educational Objective: Various international Summit in Rio de Janeiro from 3 to 14 June organisations and their mandate. 1992. It then entered into force on 21 March 11. Key: B 1994, after a sufficient number of countries had Explanation: The Climate & Clean Air Coalition ratified it. Hence option (b) is the correct answer. is the only global effort that unites governments, Educational Objective: Important international civil society and private sector, committed to treaties and their place of signing.

improving air quality and protecting the climate 9. Key: A in next few decades by reducing short-lived Explanation: Carbon fertilisation is the artificial climate pollutants across sectors. It is an enrichment of the atmosphere of greenhouses initiative of the United Nations Environment with carbon dioxide, an essential nutrient for plants and vegetables. It is also known as

www.laex.in Page No. 75 https://elearn.laex.in

n

www.laex.in

Environment UPSC

Previous Year Questions

Program and not G20, hence statement 1 is 3. Miscellaneous incorrect. 3.1 Green House Gas Protocol The Coalition's initial focus is on methane, black 13. Key: A carbon, and HFCs. Statement 2 is correct. Explanation: The Greenhouse Gas Protocol Educational Objective: Various initiatives (GHG Protocol) is the most widely used started towards combating GHG emissions. international accounting tool for government and business leaders to understand, quantify, 12. Key: B and manage greenhouse gas emissions. Hence Explanation: The ‘Global Alliance for Climate- option (a) is correct. Smart Agriculture’ was launched during the A joint initiative of UN Environment and the Climate Summit, on 23 September, 2014. Hence United Nations Development Programme, funded statement 1 is incorrect. by the Global Environment Facility (GEF)- The Global Alliance for Climate-Smart Least Developed Countries Fund, aims to Agriculture (CSA), will work for: sustainable and identify technical, institutional and financial equitable increases in agricultural productivity needs to integrate climate change adaptation and incomes; increased resilience of food into ongoing medium and long-term national systems and farming livelihoods; and reduced planning and budgeting in Least Developed greenhouse gas (GHG) emissions from Countries (LDCs) and strengthen their agriculture. The Alliance will also aim to boost institutional and technical capacities for food and nutrition security through climate- National Adaptation Plans (NAP). Option (b) refers to this Global Environment Facility adjusted and natural-resource efficient (GEF)-Least Developed Countries Fund. agricultural practices, food systems and social Option (c) refers to Kyoto Protocol. policies. It is evident that the alliance does not Educational Objective: Test candidate's impose any binding obligation on member understanding of Green house gas protocol. countries. Statement 2 is correct. The alliance is an initiative of the Food and 4. Funding Related 4.1 Green Climate Fund Agriculture Organisation (FAO) and India didn’t 14. Key: A play much role in the formation of this alliance. Explanation: The Green Climate Fund is a fund Hence statement 3 is incorrect. established within the framework of the Educational Objective: Various initiatives UNFCCC as an operating entity of the Financial taken towards mitigating the impact of climate Mechanism to assist developing countries in change. adaptation and mitigation practices to counter climate change. The GCF is based in Incheon, South Korea. Statement 1 is correct. GCF launched its initial resource mobilisation in 2014, and rapidly gathered pledges worth USD

www.laex.in Page No. 76 https://elearn.laex.in

n

www.laex.in

Environment UPSC

Previous Year Questions

10.3 billion. These funds come mainly from Educational Objective: Understand the funding developed countries, but also from some mechanism of various international bodies.

developing countries, regions, and one city. 4.3 Bio Carbon Fund Initiative

Statement 2 is incorrect as none of the 16. Key: D multilateral institutions mentioned are in any Explanation: The BioCarbon Fund Initiative for way related to GCF. Sustainable Forest Landscapes (ISFL) is a Educational Objective: Climate funds, their multilateral fund, supported by donor mandate and source of funding. governments and managed by the World Bank.

4.2 Global Environment Facility The BioCarbon Fund Initiative for Sustainable

Forest Landscapes collaborates with forest 15. Key: D countries around the world to reduce emissions Explanation: Global Environment Facility, an from the land sector through smarter land use independently operating financial planning, policies, and practices. The ISFL is organization(Hence Option (c) is incorrect), the pioneering work that enables countries and GEF provides grants for projects related to private sector actors to adopt changes in the way biodiversity, climate change, international farmers work on the ground to the way policies waters, land degradation, the ozone layer, are made at the international level. This work persistent organic pollutants (POPs), mercury, supports sustainable landscapes, climate-smart sustainable forest management, food security, land use, and green supply chains. sustainable cities. Educational Objective: Different types of funds The GEF also serves as financial mechanism associated in saving the environment and their for the following conventions: objective. 1. Convention on Biological Diversity (CBD) 2. United Nations Framework Convention on 5. Emission Reduction Climate Change (UNFCCC) 5.1REDD+ 3. United Nations Convention to Combat 17. Key: D Desertification (UNCCD) Explanation: Reducing emissions from 4. Stockholm Convention on Persistent deforestation and forest degradation (REDD+) is Organic Pollutants (POPs) a mechanism developed by Parties to the United 5. Minamata Convention on Mercury Nations Framework Convention on Climate The GEF, although not linked formally to Change (UNFCCC). It creates a financial value the Montreal Protocol on Substances that for the carbon stored in forests by offering Deplete the Ozone Layer (MP), supports incentives for developing countries to reduce implementation of the Protocol in countries emissions from forested lands and invest in low- with economies in transition. carbon paths to sustainable development. Statement (a) and (b) both are correct hence the Developing countries would receive results- right answer is (d). based payments for results-based actions. REDD+ goes beyond simply deforestation and

www.laex.in Page No. 77 https://elearn.laex.in

n

www.laex.in

Environment UPSC

Previous Year Questions

forest degradation and includes the role of diminishing forest cover and responding to conservation, sustainable management of forests climate change by a combination of adaptation and enhancement of forest carbon stocks. Hence and mitigation measures. The mission has the it covers all the options mentioned in the broad objective of both increasing the forest and question. tree cover by 5 million ha, as well as increasing Educational Objective: Learn about various the quality of the existing forest and tree cover in initiatives towards conserving the environment. another 5 million ha of forest/non forest lands

5.2 Forest Carbon Partnership Facility in 10 years. The Mission proposes a holistic view

18. Key: C of greening and focuses not on carbon Explanation: The Forest Carbon Partnership sequestration targets alone, but also, on Facility (FCPF) is a global partnership of multiple ecosystem services, especially, governments, businesses, civil society, and biodiversity, water, biomass etc., along with Indigenous Peoples focused on reducing provisioning services like fuel, fodder, timber emissions from deforestation and forest and non-timber forest produces. It will also degradation, forest carbon stock conservation, increase options of forest based livelihood of the sustainable management of forests, and the households living in the fringe of those enhancement of forest carbon stocks in developing countries, activities commonly landscapes where the Mission is implemented. referred to as REDD+. There is no financial aid Green India mission is in no way associated with to universities, individual scientists and Green accounting of Central and State budgets, institutions involved in scientific forestry nor has it got anything to do with the second research to develop eco-friendly and climate Green revolution. Hence both statements 1 and adaptation technologies for sustainable forest 2 are incorrect. management. Hence statement 2 is incorrect. Educational Objective: Green India mission Educational Objective: Various initiatives and its objectives. under REDD+. 5.4 Desertification

5.3 Green India Mission 20. Key: C

19. Key: C Explanation: Established in 1994, the United Explanation: National Mission for a Green India Nations Convention to Combat Desertification or the commonly called Green India Mission (UNCCD) is the sole legally binding international agreement linking environment and development (GIM), is one of the eight Missions outlined to sustainable land management. The under India’s action plan for addressing the Convention addresses specifically the arid, semi- challenge of climate change -the National Action arid and dry sub-humid areas, known as the Plan on Climate Change (NAPCC). GIM, drylands, where some of the most vulnerable launched in February 2014, is aimed at ecosystems and peoples can be found. protecting, restoring and enhancing India’s

www.laex.in Page No. 78 https://elearn.laex.in

n

www.laex.in

Environment UPSC

Previous Year Questions

The Convention is committed to bottom-up / Select the correct answer using the codes Decentralized approach, encouraging the given below. participation of local people in combating the a. 1, 3, 4, 6 and 7 only desertification. Hence, statement 3 is correct. b. 1, 2, 3, 5 and 6 only The secretariat helps in compiling information c. 2, 4, 5 and 7 only and reports, not in allocating finances and the d. 1, 2, 3, 4, 5, 6 and 7 south Asian region is not in focus. Hence, 1.3 Acid Rain statement 2 is wrong 3. Acid rain is caused by the pollution of Educational Objective: To know about UNCCD. environment by VIII Pollution a. Carbon dioxide and nitrogen

1. Types of Pollution b. Carbon monoxide and carbon dioxide 1.1 Plastic Pollution c. Ozone and carbon dioxide d. Nitrous oxide and sulphur dioxide 1. Why is there a great concern about the

‘microbeads’ that are released into 1.4 Photo Chemical Smog environment? 4. Photochemical smog is a resultant of a. They are considered harmful to marine the reaction among

ecosystems. a. NO2 , O3, and peroxyacetyl nitrate in the b. They are considered to cause skin cancer presence of sunlight

in children. b. CO, O2, and peroxyacetyle nitrate in the c. They are small enough to be absorbed by presence of sunlight

crop plants in irrigated fields. c. CO, CO2, and NO2, at low temperature d. They are often found to be used as food d. High concentration of NO2,O3, and CO in adulterants. the evening

1.2 E- Waste 2. Pollutants and Their Characteristics 2. Due to improper / indiscriminate 2.1 Steel Industry disposal of old and used computers or 5. Which of the following are some their parts, which of the following are important pollutants released by steel released into the environment as e- industry in India? waste? 1) Oxides of sulphur 1) Beryllium 2) Oxides of nitrogen 2) Cadmium 3) Carbon monoxide 3) Chromium 4) Carbon dioxide 4) Heptachlor Select the correct answer using the code 5) Mercury given below. 6) Lead a. 1, 3 and 4 only 7) Plutonium b. 2 and 3 only c. 1 and 4 only

www.laex.in Page No. 79 https://elearn.laex.in

n

www.laex.in

Environment UPSC

Previous Year Questions

d. 1, 2, 3 and 4 1) They are highly resistant to degradation in

2.2 Carbon Monoxide the environment. 2) They are able to accumulate in humans 6. Excessive release of the pollutant and animals. carbon monoxide (CO) into the air may Select the correct answer using the code produce a condition in which oxygen given below. supply in the human body decreases. a. 1 only What causes this condition? b. 2 only a. When inhaled into the human body, CO is c. Both 1 and 2 converted into CO2 d. Neither 1 nor 2 b. The inhaled CO has much higher affinity 2.5 Fly Ash for haemoglobin as compared to oxygen c. The inhaled CO destroys the chemical 9. With reference to ' fly ash' produced by structure of haemoglobin. the power plants using coal as fuel, d. The inhaled CO adversely affects the which of the following statements is/are respiratory center in the brain correct? 1) Fly ash can be used in the production of 2.3 In Drinking Water bricks for building construction. 7. Which of the following can be found as 2) Fly ash can be used as a replacement for pollutants in the drinking water in some some of the Portland cement contents of parts of India? concrete. 1) Arsenic 3) Fly ash is made up of silicon dioxide and 2) Sorbitol calcium oxide only, and does not contain 3) Fluoride any toxic elements. 4) Formaldehyde Select the correct answer using the code 5) Uranium given below: Select the correct answer using the codes a. 1 and 2 given below. b. 2 only a. 1 and 3 only c. 1 and 3 b. 2, 4 and 5 only d. 3 only c. 1, 3 and 5 only 3. Measures or Techniques to Reduce d. 1, 2, 3, 4 and 5

Pollution 2.4 Brominated Flame Retardants 3.1 Bioremediation

8. Brominated flame retardants are used in 10. In the context of solving pollution many household products like problems, what is/are the mattresses and upholstery, Why is there advantage/advantages of bioremediation some concern about their use? technique?

www.laex.in Page No. 80 https://elearn.laex.in

n

www.laex.in

Environment UPSC

Previous Year Questions

1) It is a technique for cleaning up pollution b. Computing oxygen levels in forest by enhancing the same biodegradation ecosystems process that occurs in nature. c. Pollution assay in aquatic ecosystems 2) Any contaminant with heavy metals such d. Assessing oxygen levels in high altitude as cadmium and lead can be readily and regions.

completely treated by bioremediation using 3.4 Miscellaneous microorganisms. 13. Consider the following; 3) Genetic engineering can be used to create 1) Carbon monoxide microorganisms specifically designed for 2) Methane bioremediation. 3) Ozone Select the correct answer using the code 4) Sulphur dioxide. given below: Which of the above are released into a. 1 only atmosphere due to the burning of crop/biomass b. 2 and 3 only residue? c. 1 and 3 only a. 1 and 2 only d. 1, 2 and 3 b. 2, 3 and 4 only c. 1 and 4 only 3.2 Air Quality Index d. 1, 2, 3 and 4 11. In the cities of our country, which

among the following atmospheric gases 14. Consider the following statements: are normally considered in calculating 1) Agricultural soils release nitrogen oxides the value of Air Quality Index? into environment. 1) Carbon dioxide 2) Cattle release ammonia into environment. 2) Carbon monoxide 3) Poultry industry releases reactive nitrogen 3) Nitrogen dioxide compounds into environment. 4) Sulfur dioxide Which of the statement given above is/are 5) Methane correct? Select the correct answer using the code a. 1 and 3 only given below. b. 2 and 3 only a. 1, 2 and 3 only c. 2 only b. 2, 3 and 4 only d. 1, 2 and 3 c. 1, 4 and 5 only Key and Explanations d. 1,2, 3, 4 and 5

1. Types of Pollution 3.3 Biological Oxygen Demand 1.1 Plastic Pollution 12. Biological Oxygen Demand (BOD) is a 1. Key: A standard criterion for a. Measuring oxygen levels in blood Explanation: Microbeads are manufactured solid plastic particles of less than one millimeter

www.laex.in Page No. 81 https://elearn.laex.in

n

www.laex.in

Environment UPSC

Previous Year Questions

in their largest dimension. They are most contribute to e-waste. Lead is used in glass frequently made of polyethylene but can be of panels and gaskets in computer monitors. other petrochemical plastics such as Cadmium occurs in SMD chip resistors, infrared polypropylene and polystyrene. These tiny pieces detectors, and semiconductor chips. of plastic are added to rinse-off products such as Beryllium is commonly found on motherboards face scrubs, toothpastes and shower gels. The and finger clips. Chromium VI (Hexavalent small spherical beads, designed to help with Chromium) is used as a corrosion protector of exfoliation and then be washed down the drain, untreated and galvanized steel plates and as a often slip through waste-water treatment plants decorative or hardener for steel housings and end up in the seas. They do not degrade Plastics. over time and can transport toxic chemicals into Mercury is used in thermostats, sensors, relays, marine organisms. Microbeads can have a switches, medical equipment, lamps, mobile damaging effect on marine life, the environment phones and batteries. and human health. This is due to their Educational Objective: To know about E composition, ability to adsorb toxins and wastes related pollution

potential to transfer it up the marine food chain. 1.3 Acid Rain Option (b): There is no evidence of microbeads 3. Key: D causing skin cancer in children. This is a made Explanation: Normally rain water has a pH of up statement, hence incorrect. 5.6 due to the presence of H+ ions formed by the Option (c): Again this is a made up statement as reaction of rain water with carbon dioxide there is no evidence of plants being able to present in the atmosphere. When the pH of the absorb microbeads. rain water drops below 5.6, it is called acid rain. Option (d): This too is a made up statement. Acid rain is a by product of a variety of human Microbeads are not used in food adulterants. activities that emit the oxides of sulphur and Educational Objective: To know about nitrogen in the atmosphere. Burning of fossil Microbeaads impact on environment fuels (which contain sulphur and nitrogenous 1.2 E- Waste matter) such as coal and oil in power stations 2. Key: B and furnaces or petrol and diesel in motor Explanation: If we observe the options carefully, engines produce sulphur dioxide and nitrogen we can see that plutonium is mentioned in three oxides. SO2 and NO2 after oxidation and of the given four options. But, Plutonium is a reaction with water are major contributors to radioactive substance and it is highly unlikely acid rain, because polluted air usually contains that it is an ingredient in making electronic particulate matter that catalyzes the oxidation. goods. If we were to eliminate this option we will 2SO2 (g) + O2 (g) + 2H2O (l) → 2H2SO4 (aq) arrive at the answer easily. 4NO2 (g) + O2 (g)+ 2H2O (l) → 4HNO3 (aq) Heptachlor is an organochlorine compound that Educational Objective: To know about Acidrain was used as an insecticide. And hence does not impact on environment

www.laex.in Page No. 82 https://elearn.laex.in

n

www.laex.in

Environment UPSC

Previous Year Questions

1.4 Photo Chemical Smog 2. Pollutants and Their Characteristics

4. Key: A 2.1 Steel Industry Explanation: Formation of photochemical smog. 5. Key: D When fossil fuels are burnt, a variety of Explanation: In steel furnace, coke reacts with pollutants are emitted into the earth’s iron ore, releasing iron and generating CO and troposphere. Two of the pollutants that are CO2 gases. Due to use of coal, pollutants such emitted are hydrocarbons (unburnt fuels) and as SOx and NOx are released, thus all are nitric oxide (NO). When these pollutants build correct options. up to sufficiently high levels, a chain reaction Educational Objective:To know about pollution occurs from their interaction with sunlight in caused by industrial sector. which NO is converted into nitrogen dioxide 2.2 Carbon Monoxide

(NO2). This NO2 in turn absorbs energy from 6. Key: B sunlight and breaks up into nitric oxide and free Explanation: Normally, oxygen would bind to oxygen atom. hemoglobin in the lungs and be released in

NO2(g) NO(g) + O(g) areas with low oxygen partial pressure (e.g. Oxygen atoms are very reactive and combine active muscles). When carbon monoxide binds to with the O2 in air to produce ozone. hemoglobin, it cannot be released as easily as

O(g) + O2 (g)  O3 (g) oxygen. The slow release rate of carbon The ozone formed in the above reaction (ii) reacts monoxide causes an accumulation of CO-bound rapidly with the NO(g) formed in the reaction (i) hemoglobin molecules as exposure to carbon to regenerate NO2. NO2 is a brown gas and at monoxide continues. Because of this, fewer sufficiently high levels can contribute to haze. hemoglobin particles are available to bind and

NO (g) + O3 (g) → NO2 (g) + O2 (g) deliver oxygen, thus causing the gradual

Ozone is a toxic gas and both NO2 and O3 are suffocation associated with carbon monoxide strong oxidizing agents and can react with the poisoning. unburnt hydrocarbons in the polluted air to Educational Objective: To know about Various produce chemicals such as formaldehyde, impact of Carbon Dioxide.

acrolein and peroxyacetyl nitrate (PAN). 2.3 In Drinking Water

7. Key: C Explanation: Contaminants are anything physical, chemical, biological, or matter in water. Drinking water may reasonably be expected to contain at least small amounts of Educational Objective: To know about basicconcepts related to photo chemical smog. some contaminants. Some drinking water contaminants may be harmful if consumed at certain levels in drinking water while others may be harmless. The presence of contaminants does

www.laex.in Page No. 83 https://elearn.laex.in

n

www.laex.in

Environment UPSC

Previous Year Questions

not necessarily indicate that the water poses a to reduce the spread of fire, but they do not health risk. make the material non-flammable. The following are general categories of drinking Polybrominateddiphenyl ethers (PBDEs) are water contaminants and examples of each: used as flame retardants  Physical contaminants primarily impact They’re persistent, bio-accumulative, and toxic the physical appearance or other physical to both humans and the environment. properties of water. Examples of physical Educational Objective: To know about contaminants are sediment or organic Brominated Flame Retardants material suspended in the water of lakes, 2.5 Fly Ash rivers and streams from soil erosion. 9. Key: A  Chemical contaminants are elements or Explanation: Statement 1 and 3 are correct. Fly compounds. These contaminants may be ash can be used in the production of bricks for naturally occurring or man-made. building construction and as a replacement for Examples of chemical contaminants some of the Portland cement contents of include nitrogen, bleach, arsenic, salts concrete.Fly ash is made up of aluminium (flouride), pesticides, metals, toxins silicate, silicon dioxide, calcium oxide. Fly ash produced by bacteria, and human or particles are oxide rich and consist of silica, animal drugs. almina, oxides of iron, calcium and magnesium  Biological contaminants are organisms in and toxic heavy metals like lead, arsenic, cobalt water. They are also referred to as and copper. Statement 3 is false as it says Fly microbes or microbiological contaminants. ash is made up of only two components namely: Examples of biological or microbial silicon dioxide and calcium oxide. contaminants include bacteria, viruses, Educational Objective: To know about fly ash protozoan, and parasites. and its impact on environment

 Radiological contaminants are chemical 3. Measures or Techniques to Reduce elements with an unbalanced number of Pollution protons and neutrons resulting in unstable 3.1 Bioremediation atoms that can emit ionizing radiation.

Examples of radiological contaminants 10. Key: C include cesium, plutonium and uranium. Explanation: Bioremediation is a process used Educational Objective: To know aboutvarious to treat contaminated media, including water, pollutants contamination in drinking water soil and subsurface material, by altering environmental conditions to stimulate growth of 2.4 Brominated Flame Retardants microorganisms and degrade the target

8. Key: C pollutants. Statement 1 is correct. Explanation: Flame retardants are used to  Toxic metals like Cadmium and Lead make it more difficult for a material to ignite or cannot be treated with this technique. Hence statement 2 is correct.

www.laex.in Page No. 84 https://elearn.laex.in

n

www.laex.in

Environment UPSC

Previous Year Questions

 It is possible to create microorganisms with Educational Objective: To know about the approach of Genetic Engineering for Biological Oxygen Demand bioremediation. 3.4 Miscellaneous Educational Objective: To know about 13. Key: D Bioremediation process to control pollution Explanation: Open biomass burning, which

3.2 Air Quality Index refers to burning of forests, savanna/grasslands

11. Key: B and crop residue, releases large amounts of Explanation: Carbon Dioxide and Methane are trace gases such as Nitrogen Oxides (NOx), CO, green house gases which are released into the methane (CH4), non-methane hydrocarbon atmosphere because of natural processes as (NMHCs), carbon dioxide (CO2) and particulate well. These gases are not air pollutants unlike matter. Carbon Monoxide, Sulfur Dioxide and Nitrogen Educational Objective: To know about biomass Dioxide which pollute the air. Hence Carbon burning and its impact on environment.

Dioxide and Methane are not part of Air Quality 14. Key: D Index. Explanation: Agricultural soils contributed to Educational Objective: To know aboutAir over 70% of N2O emissions from India in 2010, Quality Index. followed by waste water (12%) and residential

3.3 Biological Oxygen Demand and commercial activities (6%). Since 2002, N2O

12. Key: C has replaced methane as the second largest Explanation: Biochemical oxygen demand Greenhouse Gas (GHG) from Indian agriculture. (BOD) represents the amount of oxygen Statement 1 is correct. consumed by bacteria and other microorganisms  Cattle account for 80% of the ammonia while they decompose organic matter under production. Statement 2 is correct. aerobic (oxygen is present) conditions at a  The poultry industry recorded an excretion specified temperature. Even though the amount of reactive nitrogen compounds of 0.415 of dissolved oxygen in water is small, up to tonnes in 2016. That is anticipated to about ten molecules of oxygen per million of increase to 1.089 tonnes by 2030. water, it is a crucial component of natural water Statement 3 is correct. bodies; the presence of a sufficient concentration Educational Objective: To know about of dissolved oxygen is critical to maintaining the agriculture sector contribution to pollution. aquatic life and aesthetic quality of streams and lakes. Hence C is the answer.

www.laex.in Page No. 85 https://elearn.laex.in

n

www.laex.in

Environment UPSC

Previous Year Questions

www.laex.in Page No. 86 https://elearn.laex.in

n

www.laex.in